3_Matematicheskie_kruzhkix (450,5 KБ)

advertisement
Математические кружки
В этой части книги собраны материалы занятий по математике в четырех группах: двух
русскоязычных и двух англоязычных. Здесь Вы найдёте краткие конспекты занятий, содержащие
выдававшиеся ребятам задания, а также материалы математических игр, проводимых на занятиях.
Сохранены авторские названия листочков и нумерация заданий, что, мы думаем, поможет более
тонко передать внутренний мир преподавателей. Мы надеемся, что читатель осознает, что это лишь
«каркас» занятий, заполнить его полностью в рамках брошюры нереально, как нереально на словах
передать всю атмосферу живого и плодотворного общения, царившего на протяжении всей смены.
Авторы очень надеются, что предложенные материалы дадут пищу для размышлений и каждый
читатель найдёт для себя что-нибудь интересное.
Вступительный тест 15июля 2013
1. В окружность с центром О вписан треугольник АВС, у которого ∠А=20о, ∠В=45о. Найдите
∠АОВ.
2. Перечислите все известные вам формулы для нахождения площади треугольника (для
произвольных треугольников).
3. Пусть О точка пересечения диагоналей квадрата АВСD со стороной 1. Найдите длину вектора
⃗⃗⃗⃗⃗
⃗⃗⃗⃗⃗ + ⃗⃗⃗⃗⃗
𝐴𝐵 − 𝐶𝑂
𝐴𝑂 + ⃗⃗⃗⃗⃗⃗⃗
𝐵𝐷.
4. Найдите НОК (наименьшее общее кратное) чисел 105 и 777.
5. Найдите остаток от деления числа 131233+17133 на 1483.
6. Известно, что 𝑎78 − 1 делится на 223. При каких еще натуральных 𝑘 верно, что 𝑎𝑘 − 1
делится на 223.
7. Сколько среди чисел от 1 до 1000000 таких, которые дают остаток 1 при делении на 25 и
остаток 9 при делении на 16?
8. Сколько диагоналей в правильном 20- угольнике?
9. В отряде 10 человек. Сколько способов выбрать из них группу, состоящую хотя бы из 2-х
человек? (Весь отряд - тоже группа)
10. Если раскрыть скобки в выражении (𝑥 + 1)1001 , то получим
𝑥1001 + 𝑎1000 𝑥1000 + 𝑎999 𝑥 999 + … + 𝑎1 𝑥 + 𝑎𝑜 . Найдите коэффициент 𝑎3 .
11. В классе каждый мальчик переписывается с четырьмя девочками, а каждая девочка - с семью
мальчиками. Сколько детей в классе, если их от 20 до 40 человек?
12. Из 80 участников лагеря 45 знают русский язык, 28 - испанский, 73 – английский; 6
участников знают все 3 языка одновременно. Сколько участников знают хотя бы два языка?
13. Джинн бросил 2 кубика – красный и зеленый. Какова вероятность того, что сумма очков,
выпавших на них больше 8?
14. Сколько существует способов раздать 18 детям 25 карандашей и 70 ручек так, чтобы каждому
достался хотя бы один карандаш и хотя бы две ручки? Все карандаши одинаковые, все ручки
тоже. (Достаточно написать формулу, вычислять не нужно).
15. Трава на лугу растет с постоянной скоростью. Известно, что 30 коров съедают траву за 60
дней, 70 коров – за 24 дня. Сколько коров съедят всю траву ровно за 96 дней?
Вступительная Олимпиада 16 июля 2013
1) Имеется 2 ведра, объёмы которых, измеренные в литрах, - целые числа. Произведение этих
объёмов равно 2013. Всегда ли возможно набрать в неограниченно большой бак из реки ровно
1607 литров воды при помощи этих вёдер?
2) В клетках квадрата 5 х 5 расставлены числа (не обязательно целые) так, что во всех строчках
и во всех столбцах сумма чисел одинаковая. В верхнем правом квадрате 2 х 2 сумма чисел
3)
4)
5)
6)
оказалась равной 10, а в левом нижнем квадрате 3 х 3 сумма оказалась равной 15. Найдите
сумму всех чисел в квадрате.
На сторонах треугольника ABC как на основаниях, построены равносторонние треугольники:
ABC1 и A1BC -- во внешнюю сторону, а AB1C -- вовнутрь. Оказалось, что точки A1, B1, C1
лежат на одной прямой. Найдите угол ABC.
Можно ли сделать сетку 2006 на 2008 из элементов в виде буквы L, составленных из двух
единичных отрезков, так, чтобы эти фигурки не перекрывались?
Натуральные числа от 1 до 100 покрасили в три цвета. Докажите, что найдутся два
одноцветных числа, разность которых -- точный квадрат.
В квадрате 8 х 8 стоит 31 фишка: 16 на чёрных и 15 на белых клетках. Докажите, что найдутся
две соседние по стороне клетки, в которых стоят фишки.
По результатам теста и олимпиады были сформированы 4 группы, материалы занятий в
которых представлены ниже.
Материалы занятий группы English-A.
Руководители Е.Голикова и Л. Корешкова.
Задачи. Лист 1.
1.Точки M и N являются серединами сторон BC и CD параллелограмма ABCD соответственно.
Отрезки AM и BN пересекаются в точке О. Найдите отношение MO : ОА.
2.В треугольнике ABC AB = AC . На продолжении стороны AC за точку C взят отрезок CD так, что
CD = BC . Найдите углы треугольника ABC , если BD = AB.
3. а) Найдите сумму всех трёхзначных чисел, составленных из цифр 1,2,3,4 (цифры могут
повторяться).
б) Найдите сумму всех семизначных чисел, составленных из цифр 1,2,3,4,5,6 и 7 (цифры повторяться
не могут).
4.Лягушка прыгает по вершинам треугольника АВС. Каждый раз она прыгает из вершины, в которой
сидит, на одну из двух других .Сколько способов допрыгать от вершины А до вершины А используя
n ходов.
5. Числа от 1 до 13 написаны так, что каждое число является делителем суммы всех предыдущих.
а) Может ли цифра 5 быть последней в этой цепочке?
б) Какие числа могут быть на последнем месте? Докажите, что все
эти варианты возможны
и других нет.
6.В комнате лежало несколько головок сыра. Ночью крысы съели 10 из них, причём все съели
одинаковое количество сыра. После этого у некоторых крыс заболел живот. Следующей ночью
оставшиеся 7 крыс съели весь оставшийся сыр. Но каждая крыса съела в два раза меньше сыра, чем в
прошлый раз. Сколько головок сыра были в комнате до того, как пришли крысы?
7.Есть трёх и пяти центовые монеты. Можно ли заплатить любое количество центов, больше 7,
используя только эти монеты?
8. Все вершины выпуклого многоугольника окрашены в 3 цвета таким образом, что каждые две
соседние вершины окрашены в разные цвета. Докажите, что многоугольник можно разделить
диагоналями на треугольники так, что у каждого треугольника вершины будут разных цветов.
9. Существует ли такой набор из 10 целых положительных чисел, что никакие из них не делятся друг
на друга, но квадрат каждого делится на все другие.
Задачи. Лист 2.
1.Целые положительные числа А и В таковы, что 56А=65В. Докажите, что число А+В - составное.
2. а)Найти пару положительных целых чисел m и n, для которых (2an bn)n + (3ambm)m = a6b6.
b) Сколько существует пар таких чисел?
3. Числа 1, 2, 3, … , n пишут в ряд таким образом, что если где-то (не на первом месте) написано
число 𝑖, то слева от него обязательно будет хотя бы одно из чисел 𝑖 + 1 или 𝑖 − 1. Сколькими
способами можно записать строку, удовлетворяющую этим условиям, при заданном n?
4. В треугольнике ABC CD является биссектрисой прямого угла ACB; DM и DN являются высотами
треугольника ADC и BDC соответственно. Найти AC, если AM = 4, BN = 9.
5. На полке стоят 12 книг. Сколько существует способов выбрать из них 5 книг так, чтобы между
каждыми двумя из выбранных оставалась хотя бы одна невыбранная книга?
6. Доказать, что если 𝑎2 + 𝑎𝑏 + 𝑏 2 делится на 𝑎 + 𝑏, то 𝑎4 + 𝑏 4 делится на (𝑎 + 𝑏)2 .
7. Докажите, что среди 51 целых чисел есть два числа, квадраты которых дают одинаковые остатки
при делении на 100.
8. Найти все положительные целые числа n, для которых 𝑛4 + 𝑛2 + 1 является простым.
9. Есть выпуклый многоугольник с 1978 сторонами. Может ли в нём каждый угол измеряться целым
числом градусов.
Криптография – 1.
1. Найти 2004 последовательных составных положительных целых числа.
2 . Для записи текста используются только заглавные буквы, точки и пробелы – всего 28 различных
символов. Во время шифрования каждый символ был заменен на некоторое число от 0 до 27
соответственно, в порядке А, В, С,. . . , Z, точка, пробел. Затем эта последовательность чисел была
разделено на пары и каждая пара была заменена согласно правилу: пара (𝑎1 , 𝑎2 ) заменяется на пару
(𝑟28 (𝑎1 𝑛); 𝑟28 (𝑎1 𝑘 + 𝑎2 𝑚)) , где 𝑟28 (𝑥) - остаток от деления числа 𝑥 на 28 , а 𝑛, 𝑘 и 𝑚 – заранее
выбранные числа от 0 до 27. Найдите все наборы чисел 𝑛, 𝑘 и 𝑚, при которых разные пары
заменяются на разные (это необходимо для возможности расшифрования текста). Сформулируйте
правило для расшифровки в случае 𝑛 = 𝑘 = 𝑚 = 17. Решение обоснуйте.
3 . Известно, что число 14197777 равно остатку от деления числа 56887111 на некоторое число 𝑥 3 .
Числа 𝑥 и 56887111 имеют общий делитель, больший 1. Число 56887111 равно произведению двух
простых чисел. Найдите хотя бы одно возможное значение 𝑥 .
4 . Для того, чтобы попасть в секретную комнату в Хогвартсе необходимо назвать 3 целых числа 𝑎, 𝑏
и с, которые являются коэффициентами квадратной функции 𝑓(𝑥) = 𝑎𝑥 2 + 𝑏𝑥 + 𝑐. Четыре
волшебника знали следующие значения функции : Гарри Поттер 𝑓(21), Рон Уизли 𝑓(24), Драко
Малфой 𝑓(25) и Гермиона Грейнджер 𝑓(28). Когда они все собрались вместе, чтобы назвать числа
𝑎, 𝑏 и с и открыть комнату один из них назвал неправильное значение функции. Кто соврал и
почему, если Гарри назвал число 273 , Рон 357 , Драко 391, а Гермиона 497?
5 . Известно, что 10-значный номер 2013𝑥2013𝑦 делится на 121. Найти все возможные пары цифр
(𝑥, 𝑦).
6. Для получения доступа к банковской ячейке в клиенте банка Гринготтс вы должны подозвать
гоблина и сказать ему семизначный код. После первого неправильного кода гоблин взрывается.
Представьте себе, что вы не знаете код, но хотите получить доступ к некоторым ячейкам. Какое
минимальное количество гоблинов вы должны подозвать, чтобы узнать код.
7. Для зашифровки текста на английском языке его записывают в одну строку без пробелов и знаков
препинания. Заглавные буквы в нём заменяются на строчные. В получившейся строке буквы
заменяются числами в соответствии с их местом ( 1,2,3, ... , L). Шифрование производится путём
перестановки букв в строке по следующему правилу: мы берём два целых положительных числа а и
b. Буква с номером n в исходной строке после шифрования должна иметь номер, равный остатку от
деления числа an+ b на L. Если an+ b делится на L нацело, то остаток полагаем равным L. Например,
если длина строки L = 25 , а = 9 , b = 11 , то третья буква исходной строки будет на 13 месте в
зашифрованной строке ( 9 х 3 + 11 = 38, а число 38 имеет остаток 13 при деления на 25). Известно,
что после шифрования строка из 37букв
Isawhimdancingtherebytherecordmachine
Была заменена следующей строкой:
Edncraeoenirbcsdyiamtnwahghcetihrhmie.
При этих же значениях а и b была зашифрована ещё одна строка:
ooianfdrnayearoggku
Найдите значения а и b и восстановите исходную строку.
Криптография – 2.
1. На клавиатуре мобильного телефона каждой кнопке соответствует несколько букв
2-ABC, 3-DEF, 4-GHI, 5-JKL, 6-MNO, 7-PQRS, 8-TUV, 9-WXYZ. Каждой букве соответствует
некоторое количество нажатий кнопки. Например, если вы нажмете кнопку 4, один раз вы напишете
букву G. Если быстро нажать ее два раза вы напишете H, а если нажимать с паузами, то две буквы G .
Известно, что написан пароль, состоящий из 10 букв, и были нажаты кнопки 777255899999 . Найти
количество возможных паролей.
2 . Шифрование фразы было сделано в два этапа. На первом этапе каждая буква заменена следующей
в алфавите (Я заменяется А). На втором этапе простой шифр заменён с использованием секретного
ключа. Это означает, что каждая буква текста заменяется другой буквой из этого же алфавита. Ключ
представляет собой таблицу с данными о замене всех букв. Зашифрованный текст, приведен ниже:
OSZJX FXRE YOQJSZ RAYFJ
Найти оригинальный текст, если известно, что текст не меняется при перемене порядка
использования этапов. Пробелы остаются на месте.
3 . В стране 1993 города. Из каждого города идет, по меньшей мере, 93 дороги. Известно, что можно
пройти от каждого города до любого другого. Докажите, что вы сможете сделать это меньше чем за
63 перехода (переход идёт от города к городу).
4 . Можно ли поставить все 3-х значные числа не содержащие 0 на последнем месте в ряд таким
образом, чтобы последняя цифра каждого числа была первой цифрой в следующем?
5 . В Нарнии N городов, любые два из которых соединены дорогой. Ледяная Королева хочет сделать
движение по всем дорогам односторонним, таким образом, что если из города можно выехать, то в
него нельзя вернуться. Докажите, что
а) она сможет это сделать;
б) найдется город, из которого можно добраться до всех и найдется город, из которого нельзя
выехать;
в) есть только один путь, проходящий через все города;
г) сколькими способами она может это сделать?
6. Две таблицы 5х5 созданы для кодирования текста на английском языке. В ячейки каждой таблицы
буквы английского алфавита записаны так (буквы V и W считаются идентичными), что каждая буква
занимает только одну ячейку в каждой таблице. Обозначим букву 𝑖 − й строки и 𝑗 − го столбца
первой таблицы 𝑎𝑖𝑗 , второй таблицы 𝑏𝑖𝑗 . Во время кодирования текст разделен на несколько
последовательных пар. Пара 𝑎𝑖𝑗 𝑏𝑙𝑚 при 𝑖 ≠ 𝑙 заменяется парой 𝑏𝑙𝑚 𝑎𝑙𝑗 , а при 𝑖 = 𝑙 парой 𝑏𝑙𝑗 𝑎𝑖𝑚 .
После кодирования следующего текста:
сryptographicalgorithm
появились тексты:
pabdgliurcavthotueadsр,
dszquphsbqijdbmhpsjuin.
Найдите, который из них правильный и объясните почему.
7. Имеется 6 гирь весом 1,2,3,4,5,6 граммов. На каждой гире есть наклейка 1г, 2г, 3г, 4г, 5г, 6г, но они
могут не соответствовать весу гири (например, наклейка 3 может быть размещены на гире в 6 грамм).
Можно ли за два взвешивания на чашечных весах определить правильно ли расположены наклейки?
Заключительная олимпиада. Первый этап.
1. На книжной полке стоят 10 томов энциклопедии. Их номера 1, 2 , 3, . . . , 10 . Допускается
поменять местами две любые книги, с условием, что между ними, по крайней мере, четыре тома.
Всегда ли можно поставить все тома в порядке возрастания с помощью этой операции?
2 . Последовательность чисел формируется по следующему правилу: первые три его
члена 𝑎1 𝑎2 𝑎3 являются случайными целыми положительными числами. Четвертый член
𝑎4 равно остатку деления 𝑎2 𝑎3 на 𝑎1, пятое число 𝑎5 равно остатку от деления 𝑎3 𝑎4 на 𝑎2 и т.д.
Доказать, что в этой последовательности должен появиться элемент равный 0.
3 . Профессор Дамблдор решил вручить волшебные палочки ученикам. На церемонию вручения
пришли 28 учеников из Гриффиндора и 37 учеников из Слизерина. Известно, что Дамблдор дал
одинаковое количество палочек каждому гриффиндорцу и одинаковое (но может быть другое)
количество палочек каждому слизеринцу. Оказалось, что есть единственный способ дать палочки в
соответствии данным правилом (все палочки должны быть розданы). Какое наибольшее возможное
количество палочек может быть у Домблдора?
4 . 125 студентов пришли в большую аудиторию и каждый из них знал ровно десятерых других.
Некоторые из студентов покинули аудиторию во время перерыва. Известно, что все оставшиеся
студенты знают одинаковое число людей, оставшихся в зале. Докажите, что среди людей, которые
ушли, было по крайней мере два человека, знающих друг друга.
Заключительная олимпиада. Второй этап.
5 . Секретный код для сейфа ФБР представляет собой положительное целое число от 1 до 1700 . Два
шпиона обнаружили два различных кода и решили поделиться этой информацией друг с другом. Они
заранее придумали, как тайно передать друг другу данные. Они встретились на берегу рядом с
кучкой из 26 камней. Вначале первый шпион бросил в воду некоторое количество камней. Затем
второй сделал то же самое, потом снова первый и так далее, пока камни не закончились. После этого
шпионы ушли. Они не говорили и не могли обмениваться информацией каким-либо другим
способом. Итак, как же была передана информация?
6. У нас есть 8 целых чисел: 𝑎, 𝑏, 𝑐, 𝑑, 𝑒, 𝑓, 𝑔, ℎ. Доказать, что по крайней мере одно из шести чисел
𝑎𝑏 + 𝑏𝑑, 𝑎𝑒 + 𝑏𝑓, 𝑎𝑔 + 𝑏ℎ, 𝑐𝑒 + 𝑑𝑓, 𝑐𝑔 + 𝑑ℎ, 𝑒𝑔 + 𝑓ℎ -неотрицательно
7. Точка М лежит внутри равнобедренного треугольника АВС с основанием ВС. Известно, что
∠𝑀𝐵𝐶 = 300 , ∠𝑀𝐶𝐵 = 300 . Найдите ∠𝐴𝑀𝐶 если ∠𝐵𝐴𝐶 = 800 .
Материалы группы Russian-А.
Руководители И. Ибатулин, С. Бердовская.
Серия 1.
Решение линейных уравнений первой степени в целых числах
Теорема 1. Пусть даны целые числа a, b, отличные от нуля. Пусть также дано целое число с такое,
что число с кратно НОД(a; b). Если (x0; y0) является решением уравнения ax+by=c, то общий вид
решений этого уравнения записывается следующим образом
𝑏
𝑎
𝑥 = 𝑥0 +
𝑡,
𝑦 = 𝑦0 −
𝑡,
НОД(𝑎; 𝑏)
НОД(𝑎; 𝑏)
где t – целое число.
Пример 1. Решите в целых числах уравнение 7x+1739y=2048.
Теорема 2. Пусть даны взаимно простые числа a, m. Тогда полная система вычетов по
модулю m не меняется при умножении на число а.
Пример 2. Решите в целых числах уравнение 37x-705y=19.
Пример 3. Решите в целых числах уравнение 245x + 1379y=189.
Теорема 3. Пусть даны взаимно простые числа a, m. Тогда приведенная система вычетов по модулю
m не меняется при умножении на число а.
Теорема 4 (Эйлера). Пусть даны взаимно простые целые числа a и m. Тогда
𝜙(𝑚)
𝑎
≡ 1 (𝑚𝑜𝑑 𝑚) , где (m) – количество натуральных чисел, не превосходящих m и взаимно
простых с m.
Пример 4. Докажите, что среди целочисленных решений уравнения 9x+13y=1 существует
решение (𝑥0 ; 𝑦0 ) такое, что число 𝑥0 𝑦0 кратно 2190.
Теорема 5. Пусть даны целые числа a и b, отличные от нуля. Если d=НОД(a; b), то
существуют целые числа x1, y1 такие, что ax1+by1=d.
Пример 5. Решите в целых числах уравнение 156x+479y=547.
Пример 6. Решите в целых числах уравнение 13x+29y=17.
Пример 7. Решите в целых числах уравнение -25x+33y+47z-53t+29u=12.
Упражнения
1.
Докажите, что среди целочисленных решений уравнения 2013x+31y=1 существует решение
(x0; y0) такое, что число x0y0 кратно 26659.
2.
Докажите, что среди целочисленных решений уравнения 7x-18y=1 существует решение (x0; y0)
такое, что число x0y0 кратно 817.
3.
Докажите, что среди целочисленных решений уравнения 7x+19y=1 существует решение (x0; y0)
такое, что число x0y0 кратно 8001.
4.
(МГУ, филолог., 1969) Покажите хотя бы одну пара целых положительных чисел k1 и k2 таких,
что 36k1-25k2=1.
5.
Решите следующие уравнения в целых числах:
a.
b.
c.
d.
e.
f.
g.
h.
i.
j.
k.
l.
55x+89y=13.
15x+29y=10.
13x-17y=123.
335x-571y=53.
943x+769y=2013.
-7x+11y=14.
3x+5y=-73.
4x+8y=100.
13x-91y=117.
-3x+92y=-152.
6x+13y=3.
2x+19y=-145.
m.
n.
o.
p.
q.
r.
s.
t.
u.
v.
w.
x.
-5x-23y=523.
57x-49y=7.
2013x+2030z=2050.
19x+29y=39.
29x+59y=102.
1982x+1999y=2004.
2x+3y+5z=12.
4x+13y-23z=-72.
51x+95y-47z=15.
13x+35y-47z+23t=54.
11x-17y-37z-45t=234.
17x-29y+23u-33t+39u-45k=51.
y. 55x+89y=13.
z.15x+29y=10.
Серия 2.
Определение 1. Конечной цепной дробью называется число, записанное в виде
[𝑎0 ; 𝑎1 , 𝑎2 , … , 𝑎𝑠 ] = 𝑎0 +
1
𝑎1 + 𝑎2 +
1
⋱
,
1
+
1
𝑎𝑠−1 +
𝑎𝑠
где a0, a1, a2, …, as – целые числа, ak ≥ 1, k=1,2,…,s-1, as ≥ 2.
Теорема 6. Существует одна и только одна конечная цепная дробь, равная данному
рациональному числу.
Определение 2. Пусть дана конечная цепная дробь [a0; a1, a2, …, as]. n-ой подходящей дробью
(0≤n≤s) к конечной цепной дроби [a0; a1, a2, …, an] называется дробь An=[a0; a1, a2, …, an].
Теорема 7. Пусть дана конечная цепная дробь [a0; a1, a2, …, as] и конечная последовательность
Pk, Qk (k=0, 1, …, s) таких, что при n=2, 3, …, s выполнено
Pn=Pn-1an+Pn-2, Qn=Qn-1an+Qn-2,
и
P0=a0, Q0=1, P1=a0a1+1, Q1=a1.
Тогда для всякого k=0, 1, …, s выполнено 𝐴𝑘 =
𝑃𝑘
𝑄𝑘
.
Теорема 8. Пусть дана конечная цепная дробь [a0; a1, a2, …, as] и конечная последовательность
Pk, Qk (k=0, 1, …, s), определенная также как и в теореме 7. Для каждого n=1, 2, …, s
справедливо следующие соотношения
1)
PnQn-1 - Pn-1Qn = (-1)n-1;
2)
(Pn, Qn) = 1.
Теорема 9. Пусть даны целые положительные числа a и b. Если (a,b)=1, то частным
целочисленным решением уравнения (1) является
𝑥0 = (−1)𝑠 𝑐𝑃𝑠−1 , 𝑦0 = (−1)𝑠+1 𝑐𝑄𝑠−1,
𝑏
𝑃
где 𝑎 = [𝑎0 ; 𝑎1 , 𝑎2 , … , 𝑎𝑠 ], Ps-1, Qs-1  Z+ такое, что 𝑄𝑠−1 = [𝑎0 ; 𝑎1 , 𝑎2 , … , 𝑎𝑠−1 ] и
𝑠−1
1)=1.
(Ps-1;Qs-
Пример 1. Решите в целых числах уравнение 159x+413y=87.
Упражнения
1.
Запишите следующие числа в виде цепных дробей и найдите для каждого из них
подходящие дроби
a.
b.
c.
d.
e.
f.
2.
a.
b.
c.
d.
e.
931
;
g.
;
h.
;
i.
158
487
243
139
816
543
;
j.
;
k.
1027
2013
1982
958
l.
;
895
141
;
m.
;
n.
716
567
593
1114
;
o.
;
p.
;
q.
143
1249
249
871
1434
544
968
;
r.
2030
;
2012
546
;
777
91
;
1156
8961
;
7564
8961
;
7564
8551
774
.
Решите следующие уравнения в целых числах с помощью цепных дробей.
355x+789y=321;
f.
349x+957y=711;
781x+543y=214;
g.
598x+948y=179;
555x+648y=345;
h.
259x+367y=68;
496x+846y=568;
i.
58x+78y=39;
746x+1089y=512;
j.
78x+195y=272.
Добавка к сериям 1-2 тем, кто усвоил мало методов решения.
Решение с помощью алгоритма Евклида.
Пример . Решите в целых числах уравнение 156x+479y=547.
Решение. Найдем целые числа x1 и y1 такие, что
156x1+479y1=НОД(156; 479)=1.
Для этого проведём процесс из доказательства теоремы 5 для чисел 156 и 479.
В одну сторону:
479=1563+11, 156=1114+2, 11=25+1, 2=21;
в обратную сторону:
1=11-25=11-(156-1114)5=7111-1565=71(479-1563)-1565=47971+156(-218).
Следовательно, 𝑥1 = −218, 𝑦1 = 71.
Отсюда, частным целочисленным решением уравнения 156x+479y=547 является
𝑥0 = −218 ⋅ 547 = −119246,
𝑦0 = 71 ⋅ 547 = 38837.
А значит, согласно теореме 1, решением данного уравнения является
𝑥 = −119246 + 479𝑡, 𝑦 = 38837 − 156𝑡,где t – целое число.
Ответ: 𝑥 = −119246 + 479𝑡, 𝑦 = 38837 − 156𝑡, где t – целое число.
Решение с использованием теоремы Эйлера.
С помощью теоремы Эйлера в случае, когда числа a и b целые взаимно простые, можно найти
частное целочисленное решение уравнения ax+by=c по формулам :
𝑥0 = 𝑎𝜙(𝑏)−1 ⋅ 𝑐, 𝑦0 =
с−𝑎𝑥0
𝑏
, где (m) – количество натуральных чисел, не превосходящих m и
взаимно простых с m.
Метод «ловкость рук» для уравнений с несколькими переменными
Пример. Решите в целых числах уравнение -25x+33y+47z-53t+29u=12.
Решение. Положим k= - x+u, тогда
-25x+33y+47z-53t+29u=25k+33y+47z-53t+4u.
Пусть m=u+12z. Отсюда,
12=25k+33y+47z-53t+4u=25k+33y-z-53t+4m.
Следовательно,
z=25k+33y-53t+4m-12, u=m-12z=144-300k-396y+636t-47m,
x=u-k=144-301k-396y+636t-47m.
Ответ: (144-301k-396y+636t-47m; y; 25k+33y-53t+4m-12; t; 144-300k-396y+636t-47m),
где k, y, t, mZ.
ЗАДАНИЕ:
Решить уравнения в целых числах не менее чем тремя разными способами.
а) 27х – 40y = 1;
б) 54x + 37y = 7;
в) 107x + 84y =1;
г) 13x – 15y =7;
д) 81x + 52y = 5;
e) 24x – 56y = 72.
Серия 3.
Текстовые задачи и уравнения в целых числах.
1. (МГУ, эконом., отделение планирования и экономической кибернетики, 1975) Зоопарк ежедневно
распределяет 111 кг мяса между лисами, леопардами и львами. Каждой лисе полагается 2 кг мяса,
леопарду - 14 кг, Льву - 21 кг. Известно, что у каждого льва бывает ежедневно 230 посетителей, у
каждого леопарда - 160, у каждой лисицы - 20. Сколько должно быть лис, леопардов и львов в
зоопарке, чтобы ежедневно число посетителей у этих животных было максимальным?
Ответ: 1 лев, 6 леопардов, 3 лисицы
2. (МГУ, географ., 1979) В школьной газете сообщалось, что процент учащихся некоторого класса,
превысивших во втором полугодии свою успеваемость, заключен в пределах от 2,9% до 3,1%.
Определить минимальное возможное число учеников в таком классе.
Ответ: 33 ученика.
3.
(МГУ, ВМК, 1982) На заводе было несколько одинаковых прессов, штампующих детали, и
завод выпускал 6480 деталей в день. После реконструкции, все прессы заменили на более
производительные, но также одинаковые, а их число увеличилось на 3. Завод стал выпускать в день
11200 деталей. Сколько прессов было первоначально?
Ответ: 5 прессов.
4. (МГУ, физ., 1983) После деления двузначного числа на сумму его цифр в частном получается 7 и
в остатке 6. После деления этого же двузначного числа на произведение его цифр в частном
получается 3 и в остатке 11. Найдите это двузначное число.
Ответ: 83.
5 .(МГУ, эконом., отд. планирования и экономической кибернетики, 1984) Из строительных деталей
двух видов можно собрать три типа домов. Для сборки 12-квартирного дома необходимо 70 деталей
первого и 100 деталей второго вида. Для 16-квартиного дома требуется 110 и 150, а для дома на 21
квартиру нужно 150 и 200 деталей первого и второго видов соответственно. Всего имеется 900
деталей первого и 1300 деталей второго вида. Сколько и каких домов нужно собрать, чтобы общее
количество квартир в них было наибольшим?
Ответ: 1 дом на 16 квартир и 12 домов на12 квартир.
6 .(МГУ, мех-мат., 1992) Мастер делает за 1 час целое число деталей, большее 5, а ученик – на 2
детали меньше. Один мастер выполняет заказ за целое число часов, а два ученика – на один час
быстрее. Из какого количества деталей состоит заказ?
Ответ: 24.
7 .(МГУ, эконом., 1992) Цех получил заказ на изготовление 2000 деталей типа А и 14000 деталей
типа Б. Каждый из 146 рабочих цеха затрачивает на изготовление одной детали типа А время, за
которое он мог бы изготовить 2 детали типа Б. Каким образом следует разделить рабочих цеха на
две бригады, чтобы выполнить заказ за наименьшее время, при условии, что обе бригады приступят
к работе одновременно, и каждая из бригад будет занята изготовлением деталей только одного
типа?
Ответ: 33 рабочих (для изготовления деталей А) и 113 рабочих (для изготовления деталей Б).
8 .(МГУ, эконом., отделение экономики, 1996) В контейнер упакованы комплектующие изделия
трех типов. Стоимость и вес одного изделия составляет 400 тыс.руб. и 12 кг для первого типа, 500
тыс.руб. и 16 кг для второго типа, 600 тыс.руб. и 15 кг для третьего типа. Общий вес
комплектующих изделий равен 326 кг. Определите минимальную и максимальную возможную
стоимость находящихся в контейнере комплектующих изделий.
Ответ: 10500 тыс.руб. и 12600 тыс.руб.
9. (МГУ, эконом., отделение менеджмента, 1996) В контейнер упакованы комплектующие изделия
двух типов. Стоимость и вес одного изделия составляет 400 тыс.руб. и 12 кг для первого типа, 600
тыс.руб. и 15 кг для второго типа. Общий вес комплектующих изделий равен 321 кг. Определите
минимальную и максимальную возможную стоимость находящихся в контейнере комплектующих
изделий.
Ответ: 11000 тыс.руб. и 12600 тыс.руб.
10 .(МГУ, соц., 1997) В дошкольном учреждении провели опрос. На вопрос: «Что Вы
предпочитаете, кашу или компот?» - большая часть ответила: «Кашу», меньшая часть: «Компот», а
один респондент: «Затрудняюсь ответить». Далее выяснили, что среди любителей компота 30%
предпочитают абрикосовый, а 70% - грушевый. У любителей каши уточнили, какую кашу они
предпочитают. Оказалось, 56,25% выбрали манную, а 37,5% - рисовую, и лишь один ответил:
«Затрудняюсь ответить». Сколько детей было опрошено?
Ответ: 27 детей.
11 .(МГУ, мех-мат., 2001) Найдите все трехзначные числа, каждое из которых больше суммы
квадратов своих цифр на 517.
Ответ: 618, 659, 698.
12 .(МГУ, географ., 2005) В цехе имелось N одинаковых станков, которые, работая вместе,
вытачивали в день 5850 деталей. После модернизации число производимых в день каждым станком
готовых деталей возросло на 20%. Это позволило по крайне мере без сокращения общего объема
продукции цеха уменьшить число станков максимум на 4. Найдите N.
Ответ: 26.
13 .(МГУ, соц., 2005,) Студенты, проживающие в одной комнате, решили купить электрический
чайник, при этом каждый внес одинаковую сумму. Однако в последний момент один студент
отказался, и каждому из оставшихся пришлось добавить 100 рублей. Сколько студентов проживало
в комнате и какова цена чайника, если известно, что она заключена в пределах от 1000 до 1500
рублей?
Ответ: 4 студента, 1200 рублей.
Серия 4.
Модулярная арифметика.
Теорема 10 (о делении с остатком). Для любых целых чисел a и b (b0) существуют целые числа q
и r такие, что 0  r < |b| и a = bq + r.
Пример 1. Решите уравнение 3x2+1= 5y в целых числах.
Теорема 11. Для всякого целого числа n справедливы следующие соотношения
1)
n2  0, 1 (mod 3);
2)
n2  0, 1 (mod 5);
3)
n2  0, 1, 4 (mod 8);
4)
n2  0, 1 (mod 9);
5)
n2  0, 1,4, 9 (mod 16).
Пример 2 (IMO, 2003, shortlist) Найдите наименьшее целое положительное число k, для
которого существуют числа x1, x2, …, xk такие, что 𝑥13 + 𝑥23 +. . . +𝑥𝑘3 = 20022002 .
Упражнения
1.
Докажите, что указанные уравнения не имеют решений в целых числах:
2
4
а) x +3y=23;
k) 𝑥14 + 𝑥24 +. . . +𝑥14
= 1999;
2
3
3
b) x -8x+12y=51;
l) 8x -13y =17;
2
c) y +4y -11=12x;
m) 3x2=16y2+8y+5;
2
d) y +9=7x;
n) x2-1=2(4y2-1);
e) x2 +4y2=2010;
o) 2x2-5y2=7;
f) 12x+5=y2;
p) y2=3x2+8;
2
3
g) –x +7y +6=0;
q) 6x2+3x+1=2y2;
2 2 2
h) x +y +z =1999;
r) 2x2-4x-5y2-10y-10=0;
i) 15x2-7y2=9;
s) 5y2=x2+7x+9;
j) x2-5y+3=0;
t) y3=x2+x.
Метод сужения ОДЗ
Пример 2. Решите уравнение
√2 + 𝑥 − 𝑦 + √6 − 𝑥 + 𝑦 = (𝑥 + 2𝑦)2 − 2(𝑥 + 2𝑦) + 5.
Пример 3. Докажите, что уравнение 8𝑥 + 15𝑦 = 17𝑧 имеет единственное положительное
целочисленное решение x=y=z=2.
Упражнения
Решите уравнение
(𝑥 2 − 2𝑥 + 3)(𝑦 2 + 6𝑦 + 12) = 6;
√𝑥 + 2 + √6 − 𝑥 = 3𝑦 2 − 12𝑦 + 16;
𝑐𝑜𝑠2𝑥 − 2𝑦 − √𝑦 − 2𝑥 2 − 1 + 1 = 0.
𝑥(𝑦 2 + 1) = √𝑥 − 1(𝑦 − 1)2 .
(Серпинский, 1955) Найдите все целые положительные решения уравнения
3𝑥 + 4𝑦 = 5 𝑧 .
Подсказка. Рассмотрите по модулям 3, 4, 8.
3.
(Юниорская Балканская математическая олимпиада, 2012, задача 4) Найдите все целые
положительные числа x, y, z и t такие, что 2x3y+5z=7t.
4.
(Юниорская Балканская математическая олимпиада, 2001, задача 1) Решите уравнение
3
3
𝑥 + 𝑦 + 𝑧 3 = 2001 в целых положительных числах.
Подсказка. Рассмотрите по модулю 9.
5.
(Балканская математическая олимпиада, 1998, задача 4) Докажите, что следующее
уравнение не имеет решений в целых числах x2 + 4 = y5.
Рассмотрите по модулю 11.
6.
(Балканская математическая олимпиада, 2009, задача 1) Решите уравнение
x y 2
3 -5 =z в целых положительных числах.
Подсказка: рассмотрите по модулям 4, 9, 7.
1.
a.
b.
c.
d.
2.
Серия 5.
Последовательность
Определение 1. Последовательность – функция, областью определения которой
является множество целых положительных чисел.
Определение 2. Арифметической прогрессией называется такая последовательность,
в которой каждый член начиная со второго получается из предыдущего путем прибавления
одного и того же числа.
Пусть дана арифметическая прогрессия {𝑎𝑘 }+∞
𝑘=1 . Положим d=a2-a1. Согласно
определению арифметической прогрессии 𝑑 = 𝑎𝑘+1 − 𝑎𝑘 , где k=1, 2, …. Отсюда, для
натуральных n и 𝑙 имеем
𝑎𝑛+𝑙 = (𝑎𝑛+𝑙 − 𝑎𝑛+𝑙−1 ) + (𝑎𝑛+𝑙 − 𝑎𝑛+𝑙−2 ) + ⋯ + (𝑎𝑛+1 − 𝑎𝑛 ) + 𝑎𝑛 = 𝑙𝑑 + 𝑎𝑛
или
𝑎𝑚 = 𝑎𝑛 + (𝑚 − 𝑛)𝑑,
(1)
где m – натуральное число не обязательно большее n.
В случае, когда m≠n из (1) следует, что
𝑎𝑚 −𝑎𝑛
𝑑 = 𝑚−𝑛
.
(2)
Из равенства (1), если (n+m) – четно, можно также получить следующее соотношение
𝑎𝑛 +𝑎𝑚
2
=
𝑎𝑛+𝑚 𝑛−𝑚
𝑚−𝑛
+
𝑑+𝑎𝑛+𝑚 +
𝑑
2
2
2
2
2
= 𝑎𝑛+𝑚
(3)
2
или в более общем виде для l<n
𝑎𝑛 + 𝑎𝑚 = 𝑎𝑛−𝑙 + 𝑎𝑚+𝑙
(4)
+∞
На основе арифметической прогрессии {𝑎𝑘 }𝑘=1 можно построить новую
последовательность, в которой n-й член равен сумме первых n членов арифметической
прогрессии {𝑎𝑘 }+∞
𝑘=1 , т.е. 𝑆𝑛 = 𝑎1 + 𝑎2 + ⋯ + 𝑎𝑛 , где n=1, 2, ….
С помощью равенства (4) Sn представимо в виде
𝑆𝑛 =
(𝑎1 + 𝑎𝑛 ) + (𝑎2 + 𝑎𝑛−1 ) + (𝑎3 + 𝑎𝑛−2 ) + ⋯ + (𝑎𝑛 + 𝑎1 )
𝑎1 + 𝑎𝑛
=
𝑛
2
2
.
(5)
Пример 1 (МГУ, 1989, хим.). Последовательность чисел a1, a2, a3, … является арифметической
прогрессией. Известно, что a1+a5+a15=3. Найдите a5+a9.
Пример 2. Вычислите 2+5+8+…+104.
Определение 3. Геометрической прогрессией называется последовательность, в которой
первый член отличен от нуля и каждый член, начиная со второго, получается из предыдущего
путем умножения на одно и тоже число.
Пусть дана геометрическая прогрессия {𝑏𝑘 }+∞
𝑘=1 . Положим q =
аналогичные тем, которые были при доказательстве соотношений
(1)-(5), получим (n, m, l  N, l<n)
𝑏2
𝑏1
𝑏𝑚 = 𝑏𝑛 𝑞 𝑚−𝑛
𝑏𝑛 ∗ 𝑏𝑚 =
если n+m - четно
. Повторяя рассуждения
(6)
2
𝑏𝑛+𝑚
(7)
2
𝑏𝑚+𝑙 ∗ 𝑏𝑛−𝑙 = 𝑏𝑛 ∗ 𝑏𝑚
𝑇𝑛 = 𝑏1 + 𝑏2 + ⋯ + 𝑏𝑛 = 𝑏1
(8)
𝑞 𝑛 −1
𝑞−1
, где 𝑞 ≠ 0
(9)
В случае, когда |q|<1, перейдя к пределу при n в равенстве (9), получим
(10)
Пример 3 (МГУ, 1996, ВМК). Числа a, b, c, d являются последовательными членами
геометрической прогрессии. Известно, что a+d=10, ad=7. Найдите сумму b3+c3.
Упражнения
1. (МГУ, 1988, ВМК) Найдите сумму первых двадцати членов арифметической прогрессии,
если известно, что сумма третьего, седьмого, четырнадцатого и восемнадцатого ее членов
равна 10.
2. (МГУ, 2003, ВМК) Сумма первых тридцати членов геометрической прогрессии с
ненулевым первым членом и ненулевым знаменателем равна удвоенной сумме первых десяти
членов. Найдите знаменатель этой прогрессии.
3. (КФМГУ, 2001, ВМК) Сумма первых четырех членов арифметической прогрессии равна
56. Все члены этой прогрессии – натуральные числа. Двенадцатый член больше 67, но меньше
74. Найдите двадцатый член этой прогрессии.
4. (КФМГУ, 2001, ВМК) Сумма первых одиннадцати членов арифметической прогрессии
больше 203, но меньше 217. Все члены этой прогрессии – натуральные числа. Пятый член
равен 16. Найдите сумму первых пятидесяти членов этой прогрессии.
5. (КФМГУ, 2001) Укажите все возможные значения разности арифметической прогрессии,
у которой первый член на 3 больше чем разность, а произведение суммы первых трехчленов
на сумму первых четырех членов отрицательно.
6. (КФМГУ, 2004) Корни x1, x2 уравнения x2-3x+A=0 и корни x3, x4 уравнения x2-12x+B=0
образуют возрастающую геометрическую прогрессию x1, x2, x3, x4. Найдите А и В.
7. (КФМГУ, 2004) Корни x1, x2 уравнения x2-5x+A=0 и корни x3, x4 уравнения x2-9x+B=0
образуют арифметическую прогрессию x1, x2, x3, x4. Найдите А и В.
8. (МГУ, 2005, соц.) Бесконечно убывающая геометрическая прогрессия содержит член
bn=1/6. Отношение суммы членов прогрессии, стоящих перед bn, к сумме членов стоящих
после bn, равно 6. Найдите n, если сумма всей прогрессии равна 3/4.
9. (МГУ, 1970, биолог) Даны арифметические прогрессии a1, a2, a3 и b1, b2, b3. Известно, что
a1+a2+a3=b1+b2+b3, числа a1+b1, a2+b2, a3+b3 образуют геометрическую прогрессию. Доказать,
что a1=b3, a2 =b2, a3=b1.
10. (МГУ, 1971, эконом) Вычислите сумму всех натуральных чисел, не превосходящих 1000
и не делящихся на 13.
11. (МГУ, 1990, ВМК) Числа a1, a2,…, a21 образуют арифметическую прогрессию. Известно,
что сумма членов этой прогрессии с нечетными номерами на 15 больше суммы членов с
четными номерами. Найдите a12, если a20=3 a9.
12. (МГУ, 1996, мехмат) Какое наибольшее число членов может содержать конечная
арифметическая прогрессия с разностью 4 при условии, что квадрат ее первого члена в сумме
с остальными членами не превосходит 100?
13. (КФМГУ, 2003) Три числа a, b, c образуют геометрическую прогрессию, причем сумма
крайних чисел больше среднего на 3, а сумма квадратов всех чисел равна 21. Найдите
знаменатель прогрессии.
14. (КФМГУ, 2006, олимп.) Сумма первых восьми членов арифметической прогрессии равна
88, а сумма первых двух членов геометрической прогрессии равна 10. Найдите третий член
геометрической прогрессии, если ее знаменатель равен первому члену арифметической
прогрессии, а ее первый член – разности арифметической прогрессии.
15. (КФМГУ, 2003, олимп.) В геометрической прогрессии со знаменателем q первый, второй
и четвертый члены образуют арифметическую прогрессию. Найдите q, если известно, что
0<|q|<5/8.
16. (КФМГУ, 2003, олимп.) В геометрической прогрессии со знаменателем q первый, третий
и четвертый члены образуют арифметическую прогрессию. Найдите q, если известно, что
0<|q|<5/8.
17. (МГУ, 2006, мехмат, устн.) Первый член арифметической прогрессии меньше 0, сотый не
меньше 74, а двухсотый меньше 200. Количество членов этой прогрессии на интервале (0,5; 5)
ровно на два меньше, чем на отрезке [20; 24,5]. Найдите первый член и разность прогрессии.
18. (МГУ, 2006, мехмат, устн.) Первый член арифметической прогрессии меньше 0, сотый не
меньше 123, а трехсотый меньше 600. Количество членов этой прогрессии на интервале (0,5;
8) ровно на два меньше, чем на отрезке [33; 40,5]. Найдите первый член и разность
прогрессии.
19. (МГУ, 1998, социолог.) Найдите все натуральные значения параметра n, при каждом из
которых задача «Найдите арифметическую прогрессию, если известны ее девятнадцатый член
и сумма первых n членов» не имеет решений, или ее решением является бесконечное
множество арифметических проресиий.
Серия 6.
Линейные функциональные уравнения для последовательностей
Справедлива𝑎
Теорема 1. Пусть даны числа p, q. Пусть дана последовательность {𝑥𝑛 }+∞
𝑛=1 такая, что
𝑥𝑛+2 = 𝑝𝑥𝑛+1 + 𝑞𝑥𝑛 , где n=1, 2, 3, … Тогда существуют числа С1 и С2 такие, что для всякого
натурального n справедливо 𝑥𝑛 =
𝑝 ± √𝑝2 + 4𝑞
𝐶1 𝜆1𝑛 + 𝐶2 𝜆𝑛2 , где 𝜆1,2 =
2
𝑝
𝐶1 𝜆1𝑛 + 𝐶2 𝑛𝜆1𝑛 , где𝜆1 = , если 𝑝2 + 4𝑞 = 0,
2
=
𝑛
{
(𝑐1 cos(𝑛𝜑) + 𝑐2 sin(𝑛𝜑)(√−𝑞) , где sin 𝜑 =
1 𝑝2 + 4𝑞
𝑝
√
, cos 𝜑 =
, если 𝑝2 + 4𝑞 < 0
2
𝑞
2√−𝑞
Теорема 2. Пусть даны числа p, q, r. Пусть дана последовательность
такая, что
yn+2 = pyn+1 + 𝑞𝑦𝑛 + 𝑟 , где n=1, 2, 3, … Тогда последовательность 𝑥1 = 𝑦1 , 𝑥𝑚 = 𝑦𝑚 −
𝑦𝑚−1 , 𝑚 = 2,3, …, удовлетворяет равенству xk+2 = 𝑝𝑥𝑘+1 + 𝑞𝑥𝑘 , где k=1, 2, 3, …
Упражнения
1.
Найдите формулу n-го члена последовательности {𝑎𝑛 }+∞
𝑛=1 , заданной условиями 𝑎 1=2,
𝑎2=5, 𝑎𝑛 = 5𝑎𝑛−1 − 6𝑎𝑛−2 (n≥3).
2.
Найдите формулу n-го члена последовательности {𝑎𝑛 }+∞
𝑛=1 , заданной условиями 𝑎 1=0,
𝑎2=2, 𝑎𝑛 = 4𝑎𝑛−1 − 4𝑎𝑛−2 (n≥3).
3.
Найдите формулу n-го члена последовательности {𝑎𝑛 }+∞
𝑛=1 , заданной условиями 𝑎 1=3,
𝑎2 = −5, 𝑎𝑛 = √3𝑎𝑛−1 − 𝑎𝑛−2 (n≥3).
4.
Найдите формулу n-го члена последовательности {𝑎𝑛 }+∞
𝑛=1 , заданной условиями 𝑎 1=0,
𝑎2=1,𝑎𝑛 = 𝑎𝑛−1 + 2𝑎𝑛−2 + 1 (n≥3).
5.
Найдите формулу n-го члена последовательности {𝑎𝑛 }+∞
𝑛=1 , заданной условиями
2
𝑎1=𝑎2=2, 𝑎𝑛 = 𝑎𝑛−1 𝑎𝑛−2
(n≥3).
6.
Найдите формулу n-го члена последовательности {𝑎𝑛 }+∞
𝑛=1 , заданной условиями 𝑎1 =
1
1
𝑎𝑛−1 𝑎𝑛−2
, 𝑎2 = 3 , 𝑎𝑛 = 3𝑎 −2𝑎
(n≥3).
2
𝑛−2
7.
𝑛−1
Найдите формулу n-го члена последовательности {𝑎𝑛 }+∞
𝑛=1 , заданной условиями 𝑎 1=1,
an-1
an = 1+a
(n≥2).
n-1
8.
Найдите формулу n-го члена последовательности {𝑎𝑛 }+∞
𝑛=1 , заданной условиями 𝑎 1=1,
2
𝑎𝑛 = √2𝑎𝑛−1 + 1 (n≥2).9.
Найдите формулу n-го члена последовательности {𝑎𝑛 }+∞
𝑛=1 , заданной условиями 𝑎1 =
𝑎𝑛−1 −𝑎𝑛−2
𝑎2 = 1, 𝑎𝑛 = 2𝑎 −1 , (n≥3).
𝑛−1
10.
(III этап республиканской олимпиады школьников, 2007, 1 день, 9 кл)
Последовательность {𝑥𝑛 }+∞
𝑛=1 задана следующим образом: 𝑥0 = 𝑎, 𝑥1 = 2, 𝑥𝑛 = 2𝑥𝑛−1 𝑥𝑛−2 −
𝑥𝑛 − 𝑥𝑛−2 + 1 для любого 𝑛 ≥ 2 . Найдите все целые 𝑎 такие, что 2𝑥3𝑛 − 1 есть полный
квадрат для любого 𝑛 ≥ 1 .
11.
(II этап республиканской олимпиады школьников, 2007, 2 день, 10 кл) Дана
последовательность целых чисел 𝑎1 , 𝑎2 , … , 𝑎2007 , для которой справедливо 𝑎1 = 1, 𝑎2 = 3 и
при любых натуральных 2 ≤ 𝑛 ≤ 2006 выполняется равенство 𝑎𝑛+1 = 3𝑎𝑛 − 2𝑎𝑛−1. Найдите
𝑎2007 .
12.
(Заключительный этап Республиканской олимпиады школьников 2003, 1 день, 10 кл)
+∞
Заданы две последовательности {𝑎𝑛 }+∞
𝑛=1 и {𝑏𝑛 }𝑛=1 по следующему правилу 𝑎0 = 𝑏0 = 0,
2
2
𝑛
𝑎𝑛 = 𝑎𝑛−1 + 3 , 𝑏𝑛 = 𝑏𝑛−1 + 2 . Что больше 𝑎2003 или 𝑏2003 ?
Метод замены: построение новой последовательности
Теорема 3. Пусть даны число p{0, 1}, целое положительное число k, f(x) – многочлен
степени k, последовательность {𝑎𝑛 }+∞
𝑛=1 такая, что
𝑎𝑛 = 𝑝𝑎𝑛−1 + 𝑓(𝑛),
где n=2, 3, …. Тогда существует многочлен g, степень которого равна k, такой, что𝑝𝑔(𝑛 −
1) − 𝑔(𝑛) = 𝑓(𝑛) и 𝑎𝑛 = (𝑎1 + 𝑔(1))𝑝𝑛−1 − 𝑔(𝑛), где n=2, 3, …
Пример 1. Найдите последовательность {𝑎𝑛 }+∞
𝑛=1 , если
2
2
𝑎1 = 1, 𝑎𝑛 = 3 𝑎𝑛−1 + 𝑛 − 15 , где n = 2, 3, ….
Упражнения
13.
Найдите последовательность {𝑎𝑛 }+∞
𝑛=1 , если
3
2
𝑎1 = 1, 𝑎𝑛 = 2𝑎𝑛−1 + 4𝑛 + 7𝑛 + 13𝑛 − 5 , где n=2, 3, ….
14.
Найдите последовательность {𝑎𝑛 }+∞
𝑛=1 , если
𝑎1 = 1, 𝑎𝑛+1 𝑎𝑛 = 4(𝑎𝑛+1 − 1) , где n=1, 2, 3, ….
15.
Найдите последовательность {𝑎𝑛 }+∞
𝑛=1 , если
𝑎1 = 1, 𝑎𝑛+1 = 𝑎𝑛 (2 − 𝑎𝑛+1 ), где n=1, 2, 3, ….
16.
Найдите последовательность {𝑎𝑛 }+∞
𝑛=1 , если
1
𝑎1 = 1, 𝑎𝑛+1 = 16 (1 + 4𝑎𝑛 + √1 + 24𝑎𝑛 ),где n=1, 2, 3, ….
17.
(BMO, 1986, задача 3) Пусть даны числа a, b, c такие, что ab≠0 и c>0. Пусть дана
последовательность {𝑎𝑛 }𝑛≥1 заданная следующим образом: 𝑎1 = 𝑎, 𝑎2 = 𝑏 и 𝑎𝑛+1 =
Докажите, что если все члены последовательности
𝑎2 +𝑏 2 +𝑐
2 +𝑐
𝑎𝑛
𝑎𝑛−1
.
являются целыми тогда и только
тогда, когда 𝑎, 𝑏, 𝑎𝑏 ∈ 𝑍 .
18.
(BMO, 1990, задача 1) Пусть дана последовательность {𝑎𝑛 }𝑛≥1 , определенная
следующим образом: 𝑎1 = 1, 𝑎2 = 3, и
𝑎𝑛+2 = (𝑛 + 3)𝑎𝑛+1 − (𝑛 + 2)𝑎𝑛 , n=1, 2, …. Найдите все значения n, при которых число
𝑎𝑛 кратно 11.
19.
(Греция, 2001, BMO, 2004, задача 1) Пусть дана последовательность {𝑎𝑛 }𝑛≥1 ,
определенная следующим образом: 𝑎1 = 3 , для любых целых неотрицательных m ≥ n
выполнено равенство
𝑎2𝑚 + 𝑎2𝑛
𝑎𝑚+𝑛 + 𝑎𝑚−𝑛 − 𝑚 + 𝑛 − 1 =
2
Найдите 𝑎2004 .
20.
(Босния и Герцеговина, отбор национальной команды 2013) Последовательность{𝑎𝑛 }
определена следующим образом: 𝑎0 = 𝑎1 = 1 и 𝑎𝑛+1 = 14𝑎𝑛 − 𝑎𝑛−1 − 4, 𝑛 ≥ 1 . Докажите,
что все члены последовательности {𝑎𝑛 } являются полными квадратами.
21.
(Болгария, 1999) Последовательность{𝑎𝑛 } определена следующим образом: 𝑎0 = 𝑎1 =
1 и (𝑛 − 1)𝑎𝑛+1 = (𝑛 + 1)𝑎𝑛 − 2𝑛 + 2, n≥1. Если 2000|𝑎1999 , то найдите наименьшее n ≥ 2
такое, что 2000|𝑎𝑛 .
22.
(Греция, 2013) Пусть дана последовательность
, определенная следующим
𝑛+1
образом: 𝑎1 = 2, 𝑎𝑛 = (𝑛−1) (𝑎1 + 𝑎2 + ⋯ + 𝑎𝑛−1 ) , n≥2. Найдите 𝑎2013 .
23.
(BMO, 2008, задача 4) Пусть дано целое положительное число с. Последовательность
𝑎1 , 𝑎2 , … определена следующим образом:𝑎1 = 𝑐, 𝑎𝑛+1 = 𝑎𝑛2 + 𝑎𝑛 + 𝑐 2 , n=1, 2, 3, …. Найдите
такие число с, что для некоторого k≥1 и некоторого m≥2 число 𝑎𝑘2 + 𝑐 2 является m-ой
степенью некоторого целого числа.
24.
(BMO, 2009, задача 4) Пусть S – множество целых положительных чисел. Найдите все
функции f: SS такие, что
𝑓(𝑓 2 (𝑚) + 2𝑓 2 (𝑛)) = 𝑚2 + 2𝑛2 , где m, n ∈ S.
Листок 8
Классические неравенства
Номер
неравенства
1
Название, формулировка
𝑛
∑ 𝑎𝑘 ≥ 𝑛,
𝑘=1
где a1, a2, …, an≥0 и
a1a2…an=1.
2
Неравенство Коши
𝑛
𝑛
𝑛
∑ 𝑥𝑘 ≥ 𝑛 ⋅ √∏ 𝑥𝑘 ,
𝑘=1
3
𝑘=1
Идея доказательства
Повторить несколько раз процедуру
замены набора вида A=(a1, a2, …, an), где
a1a2…an=1, an-1<1<an, на набор (b1, b2,
…, bn), в котором b1=a1, b2=a2, …, bn-2=an2, bn-1=an-1an, bn=1. Доказав при этом, что
из неравенства an-1+an>an-1an+1 следует,
что ∑𝑛𝑘=1 𝑎𝑘 ≥ ∑𝑛𝑘=1 𝑏𝑘 . В итоге мы
получим набор, в котором все элементы
равны 1. Именно при этом наборе
достигается равенство.
Следствие
неравенства
(1),
если
положить
𝑥𝑗
𝑎𝑗 = 𝑛
,
√𝑥1 ⋅ 𝑥2 ⋅ … ⋅ 𝑥𝑛
Где j=1, 2, …, n.
где x1, x2, …, xn≥0.
AM-HM
(неравенство Применяя неравенство (2), получим
между
средним
𝑛
𝑛
𝑛
1
1
арифметическим и средним
√∏
∑
≥
𝑛
,
гармоническим)
𝑥
𝑥
𝑘
𝑘
𝑛
𝑘=1
𝑘=1
1
𝑛2
∑ ≥ 𝑛
,
𝑛
𝑛
𝑥𝑘 ∑𝑘=1 𝑥𝑘
𝑛
𝑘=1
∑ 𝑥𝑘 ≥ 𝑛 √∏ 𝑥𝑘 ,
где x1, x2, …, xn>0.
𝑘=1
4
5
𝑘=1
откуда, следует требуемое.
(𝑎 − 𝑏)2 + (𝑏 − 𝑐)2 + (𝑐 − 𝑎)2 ≥ 0
𝑎2 + 𝑏 2 + 𝑐 2 ≥
≥ 𝑎𝑏 + 𝑏𝑐 + 𝑐𝑎,
где a, b, cR.
Неравенство Коши-Шварца Рассмотрите функцию
𝑛
(Коши-Буняковского)
2
𝑛
𝑓(𝑡) = ∑(𝑥𝑘 𝑡 − 𝑦𝑘 )2 ,
𝑘=1
(∑ 𝑥𝑘 𝑦𝑘 ) ≤
которая
всюду
принимает
𝑘=1
неотрицательные
значения.
𝑛
𝑛
𝐵
Следовательно, 𝑓 (𝐴) ≥ 0.
(∑ 𝑥𝑘2 ) (∑ 𝑦𝑘2 ),
𝑘=1
𝑘=1
≤
где x1, x2, …, xn, y1, y2, …, yn
R.
𝑛
Следствие неравенства (5)
𝑎𝑘2 (∑𝑛𝑘=1 𝑎𝑘 )2
∑
≥
,
∑𝑛𝑘=1 𝑏𝑘
𝑏𝑘
6
𝑘=1
где b1, b2, …, bn≥0, a1, a2, …,
an R.
7
𝑛
𝑛
𝑘=1
𝑘=1
𝑛
Следствие неравенства (5)
𝑎𝑘
(∑ ) (∑ 𝑎𝑘 𝑏𝑘 ) ≥
𝑏𝑘
2
≥ (∑ 𝑎𝑘 ) ,
𝑘=1
где b1, b2, …, bn, a1, a2, …,
an ≥0.
Неравенство Чебышева
После раскрытия скобок, приведения
𝑛
𝑛
подобных и некоторой группировки
(∑ 𝑎𝑘 ) (∑ 𝑏𝑘 ) ≤
исходное неравенство запишется в виде
8
𝑘=1
𝑛
𝑘=1
≤ 𝑛 (∑ 𝑎𝑘 𝑏𝑘 ),
∑ (𝑎𝑖 − 𝑎𝑗 )(𝑏𝑖 − 𝑏𝑗 ) ≥ 0.
𝑘=1
1≤𝑖<𝑗≤𝑛
где b1, b2, …, bn, a1, a2, …,
an R, (ai-aj)(bi-bj)≥0,
i, j = 1, 2, …, n.
(a+b+c)2≥3(ab+bc+ca),
Следствие неравенства (4)
где a, b, cR.
9
Упражнения
Докажите следующие неравенства
1.
ab+bc+ca≥9abc, где a+b+c=1, a, b, c>0;
2.
(𝑎 + ) (𝑏 + ) (𝑐 + ) ≥ 8, где a, b, c>0;
3.
4.
5.
6.
1
1
1
𝑏
𝑐
𝑎
𝑎𝑏 2
9𝑎𝑏𝑐
𝑏𝑐 2
𝑐𝑎2
≤ 𝑎+𝑏 + 𝑏+𝑐 + 𝑐+𝑎 ≤
2(𝑎+𝑏+𝑐)
𝑎2 +𝑏2
𝑐
+
𝑏 2 +𝑐 2
𝑎
+
𝑐 2 +𝑎2
𝑏
𝑎2 +𝑏2 +𝑐 2 +𝑎𝑏+𝑏𝑐+𝑐𝑎
√𝑎+√𝑏+√𝑐
𝑎2 +𝑏2
𝑎+𝑏
+
𝑏 2 +𝑐 2
𝑏+𝑐
+
𝑎2 +𝑏2 +𝑐 2
2
, где a, b, c>0;
≥ 2, где a+b+c=1, a, b, c>0;
≥ 2, где abc=1, a, b, c>0;
𝑐 2 +𝑎2
𝑐+𝑎
≥ 𝑎 + 𝑏 + 𝑐, где a, b, c>0;
7.
8.
𝑎
𝑏+2𝑐
𝑎
𝑏+1
+
𝑏
𝑐+2𝑎
𝑏
+
𝑐
𝑎+2𝑏
𝑐
+ 𝑐+1 + 𝑎+1 ≥
≥ 1, где a, b, c>0;
3(𝑎+𝑏+𝑐)
3+𝑎+𝑏+𝑐
, где a, b, c>0;
1
1
1
9.
a+b+c ≥ ab+bc+ca, где a, b, c>0: 𝑎+𝑏+1 + 𝑏+𝑐+1 + 𝑐+𝑎+1 ≥ 1;
10.
2(a8+b8)≥(a3+b3) (a5+b5) , где a, b >0;
11.
12.
𝑎3 +𝑏3 +𝑐 3
𝑏 3 +𝑐 3 +𝑑3
+ 𝑏+𝑐+𝑑 +
𝑎+𝑏+𝑐
где a, b, c, d>0.
𝑐 3 +𝑑3 +𝑎3
𝑐+𝑑+𝑎
+
𝑑3 +𝑎3 +𝑏3
𝑑+𝑎+𝑏
≥ 𝑎2 + 𝑏2 + 𝑐 2 + 𝑑2 ,
(a+b+c)3 ≥ a3+b3+c3+24abc, где a, b, c>0.
Метод баланса коэффициентов
13.
𝑎𝑏𝑐
(1+𝑎)(𝑎+𝑏)(𝑏+𝑐)(𝑐+16)
1
14.
a6+b6+c6+d6 ≥ abcd(ab+bc+cd+da), где a, b, c, d>0.
15.
(a3+b)(b3+c) (c3+a) ≥ 125abc, где a, b, c ≥2.
16.
10a2+10b2+c2 ≥ 4, где a, b, c>0, ab+bc+ca=1.
≤ 81, где a, b, c>0.
Теорема 1. Пусть дано положительное число k. Тогда
min
(𝑘(𝑥 2 + 𝑦 2 ) + 𝑧 2 ) =
𝑥𝑦+𝑦𝑧+𝑧𝑥=1,
𝑥,𝑦,𝑧>0.
17.
−1 + √1 + 8𝑘
.
2
5a2+4b2+5c2+d2≥2√2, где a, b, c, d>0, ab+bc+cd+da=1.
Сравнение двух походов в комбинаторике
Первый (классический) подход. Производится выборка из n элементов, которые
раскладываются на k мест (n≥k). Все места должны быть использованы, на одном месте может
быть только один элемент (естественно возникает из понятия выборки). Задача заключается в
подсчете количества всевозможных выборок определенного типа (упорядоченных,
неупорядоченных, с повторениями, без повторений) из n элементов по k.
Второй подход. Производится выборка из k частиц, которые раскладываются в n ячеек
(ящиков) (n≥k). В каждой ячейке может находиться 0, 1, 2, … частиц. Задача заключается в
подсчете количества всевозможных способов разложения k частиц (различимых и
неразличимых) в n ячеек (с и без ограничений на число попавших в каждую ячейку частиц).
Количество упорядоченных выборок
(размещений) из n элементов по k с
повторениями
Количество способов, с помощью
которых можно заполнить n разных
ячеек k различимыми частицами без
ограничений на число попавших в
каждую ячейку частиц,
𝒌 = 𝒏𝒌
А̃
𝒏
Пример 1. Найдите количество
трехбуквенных «слов», которые могут быть
составлены из 10 букв алфавита.
Пример 1. Найдите количество различных
способов покупки 3 билетов Бинго у 10
агентов.
Количество упорядоченных выборок
(размещений) из n элементов по k без
повторений (n≥k )
Количество способов, с помощью
которых можно заполнить n разных
ячеек k различимыми частицами, причем
в каждой ячейке может находиться не
более одной частицы (n≥k)
𝑨𝒌𝒏 =
Пример 2. Найдите количество различных
способов
извлечения
5
карточек
(извлеченные карточки располагаются
слева направо в порядке извлечения) из 33
карточек
с
написанными
буквами
казахского алфавита.
Количество неупорядоченных выборок
(сочетаний) из n элементов по k без
повторений (n≥k)
𝒏!
(𝒏 − 𝒌)!
Пример 2. Сколькими способами 5
студентов могут рассесться в 33-ти вагонах
поезда Астана-Алматы, так, чтобы никакие
два студента не сидели в одном вагоне.
ячеек k неразличимыми частицами,
причем в каждой ячейке может
находиться не более одной частицы,
равно числу сочетаний из n элементов по
k элементов (n≥k)
Количество
способов,
с
помощью
которых можно заполнить n различных
𝑪𝒌𝒏 =
Пример 3. Найдите количество способов
вытащить 3 теннисных мячика из набора, в
котором 52 мяча.
𝒏!
𝒌! (𝒏 − 𝒌)!
Пример 3. На склад привезли три
одинаковых телевизора. Зав. складом
провел инвентаризацию и определил, что в
секции «бытовая техника» имеется 52
свободных места (на одном месте может
находиться только один телевизор).
Определите
количество
способов
расположения телевизоров на свободные
места.
Количество неупорядоченных выборок
(сочетаний) из n элементов по k с
повторениями
Количество
способов,
с
помощью
которых можно заполнить n различных
ячеек k неразличимыми частицами без
ограничения на число попавших в
каждую ячейку частиц
(𝒏 + 𝒌 − 𝟏)!
̃𝒌 = 𝑪𝒌
𝑪
=
𝒏
𝒏+𝒌−𝟏
𝒌! (𝒏 − 𝟏)!
Пример 4. Билет на поезд, состоящий из 33 вагонов, были выставлены на продажу. Известно,
что в течение первых 15 минут были проданы 5 билетов. Кассир в отчете записывает только
номера вагонов в порядке возрастания. Определите количество возможных различных записей
кассира.
Пример 4. В 33 ящика случайным образом кладут 5 одинаковых теннисных мячика.
Определите
количество
способов
расположения
данных
мячей
в
ящиках.
Таким образом, не вызывает сомнений, что при решении комбинаторных задач ВСЕГДА
необходимо понять к какому именно подходу она относится и только, потом применять
формулы, а не сначала применять формулу, и затем сверяться с ответом.
1.
Сколько существует треугольников, у которых длина каждой стороны принимает
одно из значений 4, 5, 6, 7?
2.
Сколько различных слов можно получить, переставляя буквы слова а)
«математика»; б) «парабола»; в) «ингредиент».
3.
Необходимо составить команду космического корабля из трех человек: командира,
инженера и врача. На место командира имеется четыре кандидата: a1, a2, a3, a4; на место
инженера – три кандидата: b1, b2, b3 и на место врача – тоже три кандидата: с1, с2, с3.
Проведенная проверка показала, что командир a1 психологически совместим с
инженерами b1 и b3 и врачами с2, с3; командир a2 – с инженерами b1 и b2 и всеми врачами;
командир a3 – с инженерами b1 и b2 и врачами с1, с3; командир a4 – со всеми инженерами и
врачом с2. Кроме того, инженер b1 психологически совместим с врачом с3, инженер b2 – с
врачом с1 и инженер b3 – с врачом с2. Сколькими способами при этих условиях может
быть составлена команда корабля?
4.
Сколько слов, содержащих 6 букв, можно составить из 33 букв русского алфавита
при условии, что любые две стоящие буквы различны (например, слово «корова»
допускается, а слово «колосс» нет)?
5.
Из Лондона в Брайтон ведут 2 шоссе, соединенные 10 проселочными дорогами.
Сколькими способами можно проехать из Лондона в Брайтон так, чтобы дорога не
пересекала себя?
6.
Сколькими способами можно выбрать из полной колоды карт, содержащей 52
карты, по одной карте каждой масти? А если среди вынутых карт нет ни одной пары
одинаковых, т.е. двух королей, двух десяток и т.д.?
7.
Сколькими способами можно выбрать из полной колоды карт, содержащей 52
карты, по одной карте каждой масти так, чтобы карты красных мастей и черных мастей
образовывали пары (например, девятка пик и треф и валет бубен и червей). А так, чтобы
из выбранных карт можно было составить две пары, состоящие из черной и красной карт
одного и того же названия (например, валет пик и червей и дамы треф и бубен)?
8.
Сколькими способами из 28 костей домино можно выбрать две кости так, чтобы их
можно было приложить друг к другу?
9.
Сколькими способами можно поставить на доску две шашки – белую и черную так,
чтобы белая шашка могла бить черную?
10.
В почтовом отделении продаются открытки 10 видов. Сколькими способами
можно купить в нем 12 открыток? Сколькими способами можно купить 8 открыток?
Сколькими способами можно купить 8 различных открыток?
11.
Имеется три курицы, четыре утки и два гуся. Сколькими способами можно выбрать
из них несколько птиц так, чтобы среди выбранных оказались и куры, и утки, и гуси?
12.
Укротитель хищных зверей хочет вывести на арену цирка 5 львов и 4 тигров; при
этом нельзя, чтобы два тигра шли друг за другом. Сколькими способами он может
расположить этих зверей?
13.
Сколькими способами можно переставить буквы слова «перемет» так, чтобы три
буквы «е» не шли подряд.
14.
Найдите сумму четырехзначных чисел, получаемых при всевозможных
перестановках следующих 4 цифр: а) 1, 2, 3, 4; б) 1, 2, 2, 5; в) 1, 3, 3, 3; г) 1, 1, 4, 4.
Листок для тех, мимо кого прошли теоремы Чевы и Менелая.
Теорема Чевы.
Отрезки, соединяющие вершины треугольника с точками на противолежащих сторонах,
называют чевианами.
Теорема. Пусть в  ABC на сторонах BC,AC и AB или их продолжениях взяты
соответственно точки A 1 ,B 1 и C 1 , не совпадающие с вершинами треугольника. Прямые
AA 1 , CC 1 и BB 1 пересекаются в одной точке или параллельны тогда и только тогда,
когда выполняется равенство
AC1 BA1 CB1
.
.
=1
C1 B A1C B1 A
(1)
Примечание : процедура составления (1) не зависит от выбора «отправной» вершины
и направления обхода, так как всегда будет получаться произведение, равное 1.
AC1 BA1 CB1
CA1 BC1 AB1 1
Действительно, если
.
.
=m=1, то
= =1 (и т.д.).


C1 B A1C B1 A
A1 B C1 A B1C m
В каждом из рассмотренных случаев – и в случае внутренней точки O и в случае внешней
точки O- условие (1) Чевы можно записать также в виде
sin ACC1 sin CBB1 sin BAA1
.
.
=1
sin BCC1 sin ABB1 sin CAA1
(2)
1.3.Теорема Менелая.
Пусть на сторонах AB,BC и на продолжении стороны AC (либо на продолжениях сторон
AB,BC и AC)  ABC взяты соответственно точки C 1 ,A 1 и B 1
, не совпадающие с вершинами  ABC. Точки A 1 , B 1 ,C 1
лежат на одной прямой тогда и только тогда, когда
выполняется равенство
AC1 BA1 CB1
.
.
=1
C1 B A1C B1 A
(3)
Равенство Менелая аналогично условию Чевы, и его можно записывать, начиная с любой
вершины треугольника в любом направлении ( по часовой стрелке, против часовой стрелки).
Легко заметить, что при составлении равенства надо переходить от вершины к вершине
через точку пересечения секущей линии с этой стороной или ее продолжением; заканчивать
необходимо в той же вершине, с которой начали.
Обозначим F=
AC1 BA1 CB1
.
.
.Замечаем, что утверждение F=1 имеет место и в теореме
C1 B A1C B1 A
Чевы, и в теореме Менелая. Поэтому справедливо следующее утверждение:
Пусть в треугольнике ABC на прямых AB,BC и AC взяты точки C 1 ,A 1 ,B 1 , причем k из них
лежат на сторонах треугольника и 3 - k - на продолжениях сторон. Тогда
а) точки A 1 ,B 1 ,C 1 лежат на одной прямой тогда и только тогда, когда F=1 и k четно
(теорема Менелая);
б) прямые AA 1 , BB 1 и СС 1 пересекаются в одной точке или параллельны тогда и только
тогда, когда F=1 и k нечетно (теорема Чевы).
Задача 1.В треугольнике ABC на стороне AC взята точка N так, что AN : NC = m : n, на
стороне BC- точка K . BN пересекает AK в точке Q, BQ : QN= p:q. Найти отношение
площадей треугольников AKC и ABK.
2 способа решения:
Решение 1) AN:NC=m : n,
BQ:QN= p:q, S AKC : S ABK  KC : BK ( т.к. высоты равны)
2) Дополнительное построение: ND || BC.
3) AKC ~ AMN по двум углам 
KC AC m  n
MN (m  n)
;


 KC 
MN NA
m
m
BK BQ p
p

  BK  MN  ;
MN QN q
q
KC MN (m  n)q (m  n)q
( m  n) q


.
, S AKC : S ABK 
BK
m  MN  p
mp
mp
4)  MQN~  KQB по двум углам 
Ответ:
( m  n) q
mp
РЕШЕНИЕ 2
Рассмотрим  BCN и секущую AK, K BC , Q  BN , A  NC. Тогда по теореме Менелая
CK BQ NA
CK p m
CK (m  n)q
CK (m  n)q
; S AKC : S ABK 


 1;
 
 1;

.

KB QN AC
KB q m  n
KB
mp
KB
mp
Задача 2. AM=
2
1
AC ; BK  BC. В каком отношении отрезок BM делит отрезок AK?
5
3
AC1 : C1B  2 :1, BA1 : AC
 1: 3. Найти CB1 : B1 A .
1
Задача 3.
Задача 4. В треугольнике ABC на сторонах AB и BC взяты точки K и M , KM  AC= P.
Найти CP: AP, если а) AK: KB= 2, BM: MC= 1:3;
б) AK: KB= 3, BM: MC = 4;
в) AK: KB= 2:5, BM: MC = 2.
Задача 5. В треугольнике ABC на сторонах AB и BC взяты точки K и M; CK  AM=O.
Обозначим AK : KB= k, BM : MC= m, CO: OK= p, AO : OM= l. Пусть из четырех чисел k,m,p
и l известны два. Найти два оставшихся числа, если
1
а) k=2, m= ;
3
б) k=
в) k=3, p= 2;
2
1
, m= ;
3
2
г) k=2, l=3;
1
1
д) m= , l= ;
3
3
е) p=2, l=1.
Задача 6 . В  ABC на сторонах AB, BC,AC взяты точки C 1 , A 1 ,B 1 соответственно. Отрезки
BB 1 ,AA 1 , CC 1 пересекаются в точке O. CB 1 : B 1 A=p, CA 1 : A 1 B=q.
Найти :
АО
ОА1
;
ОВ
ОВ1
;
СО
ОС1
;
АС1
ВС1
.
Задача 7. В каком отношении делит сторону BC треугольника ABC прямая, проходящая
через точку A и середину медианы, выходящей из B?
Задача 8. В треугольнике АВС проведены биссектриса АА1 и медиана ВВ1. Известно, что
АВ=2 , АС=3. Найдите отношение ВО : ОВ1.
Задача 9. На стороне NP квадрата MNPQ взята точка A, а на стороне PQ – точка B так,
что NA:AP = PB:BQ = 2:3. Точка L является точкой пересечения отрезков MA и NB. В
каком отношении точка L делит отрезок MA?
Задача 10. Дан параллелограмм ABCD. Точка M делит отрезок AD в отношении p , а
точка N делит отрезок DC в отношении q. Прямые BM и AN пересекаются в точке S.
Вычислите отношение AS:SN.
Задача 11. На сторонах AС и BC ABC взяты точки М и L так, что AM:MC=4:1,
CL:LB=3:1. Отрезки AL и BM пересекаются в точке Q. S BLQ  1 .
1) Найти S ABC ; 2) На АВ взяли точку N так, что CN – медиана ; CN  BM  P ,
CN  AL  R . Найти S PQR .
Задача 12. На сторонах АВ, ВС и СА треугольника АВС отмечены точки C1 , A1 и B1
соответственно так, что отрезки AA1 , BB1 и CC1 пересекаются в одной точке Q,
расположенной внутри треугольника АВС. Пусть Р – точка пересечения отрезков A1C1 и
BB1 . Доказать, что
BQ
BP
.
2
QB1
PB1
Задача 13. В трапеции ABCD с основаниями AD и BC через точку A проведена прямая,
которая пересекает диагональ BD в точке E и боковую сторону CD в точке K, причем
BE:ED=1:2, CK:KD=1:4. Найдите отношение длин оснований трапеции.
Листок 11
Геометрические задачи Международной Жаутыковской олимпиады
1.
(МЖО, 2008, задача 1) Пусть K, L, M, N середины сторон AB, BC, CD, DA
выпуклого четырехугольника ABCD соответственно. Прямая KM пересекает диагонали AC
и BD в точках P и Q соответственно. Прямая LN пересекает диагонали AC и BD в точках R
и S соответственно. Докажите, что если APPC=BQQD, то ARRC=BSSD.
2.
(МЖО, 2011, задача 1) Пусть дана трапеция ABCD, пусть также M, N – середины
оснований AD и BC соответственно.
a.
Докажите, что трапеция является равнобедренной, если точка пересечения
серединных перпендикуляров к боковым сторонам трапеции лежит на отрезке MN.
b.
Остается ли утверждение пункта а справедливым, если известно только, что точка
пересечения серединных перпендикуляров к боковым сторонам трапеции лежит на
прямой MN.
3.
(МЖО, 2012, задача 1) Пусть дан остроугольный треугольник АВС, пусть D –
произвольная внутренняя точка отрезка АВ. Пусть также M и N – основание
перпендикуляров, опущенных из точки D на ВС и АС соответственно, а H1 и H2 –
ортоцентры треугольников MNC и MND соответственно. Докажите, что площадь
четырехугольника AH1BH2 не зависит от положения точки D на АВ.
4.
(МЖО, 2006, задача 2) На сторонах АВ и АС треугольника АВС отмечены точки K и
L таким образом, что ВК=CL. Пусть Р – точка пересечения отрезков BL и CK, М –
внутренняя точка отрезка АС такая, что МР параллельна биссектрисе угла ВАС. Докажите,
что СМ=АВ.
Дополнительное построение: N=BAPM.
Подсказка: теорема Менелая для треугольника АКС, линий МР и BL.
5.
(МЖО, 2007, задача 2) Внутри четырехугольника ABCD выбрана точка М такая,
что MBC=MDC, MBA=MCD. Докажите, что угол ADC равен углу ВМС или углу
АМВ, если известно, что ВАС=DAC.
Дополнительное построение: N – точка пересечения прямой АВ и описанной окружности
треугольника ВМС.
6.
(МЖО, 2010, задача 2) Пусть дан вписанный четырехугольник ABCD такой, что
AB=AD. На сторонах ВС и CD четырехугольника ABCD выбраны точки N и M таким
образом, что MN=BM+DN. Пусть прямые AM и AN пересекают описанную окружность
четырехугольника ABCD в точках P и Q соответственно. Докажите, что ортоцентр
треугольника APQ лежит на отрезке MN.
7.
(МЖО, 2008, задача 5) Не пересекающиеся окружности 1 и 2 с центрами O1 и O2
касаются прямой l в точках A1 и А2 соответственно (окружности лежат по одну сторону от
прямой l). Пусть К – середина отрезка A1А2. На окружностях 1 и 2 выбраны точки В1 и
В2 таким образом, что КВ1 и КВ2 касаются окружностей 1 и 2 соответственно (точка А1
отлична от В1, точка А2 отлична от В2). Прямые А1В1 и А2В2 пересекаются в точке L, а
прямые KL и O1O2 пересекаются в точке Р. Докажите, что точки В1, В2, Р, L лежат на
одной окружности.
8.
(МЖО, 2009, задача 5) Пусть дан четырехугольник ABCD такой, что B=D. На
отрезке АВ выбрана точка М таким образом, что AD=AM. Пусть N – точка пересечения
прямых DM и CB. Пусть H и K – основание перпендикуляров, опущенных из точек D и C
на прямые АС и AN соответственно. Докажите, что MHN=MCK.
9.
(МЖО, 2012, задача 5) На диагоналях выпуклого четырехугольника ABCD
построены равносторонние треугольники ACВ и BDC таким образом, что точки В и В
лежат по одну сторону от АС, и точки С и С лежат по одну сторону от BD. Найдите
BAD+CDA, если BC=AB+CD.
10.
(МЖО, 2006, задача 6) Пусть дан выпуклый шестиугольник ABCDEF такой, что
AD=BC+EF, BE=AF+CD, CF=DE+AB. Докажите, что
𝐴𝐵
𝐶𝐷
𝐸𝐹
= 𝐴𝐹 = 𝐵𝐶.
𝐷𝐸
Подсказка.
Докажите
с
помощью
векторов
неравенство
(AB+DE)2+(AF+CD)2+(BC+EF)2≥AD2+BE2+CF2
11.
(МЖО, 2007, задача 6) Диагонали AD, BE, CF выпуклого шестиугольника ABCDEF
пересекаются в точке М. Известно, что треугольники АВМ, ВСМ, CDM, DEM, EFM, FAM
являются остроугольными и центры их описанных окружностей лежат на одной
окружности. Докажите, что четырехугольники ABDE, BCEF, CDFA имеют одинаковую
площадь.
12.
(МЖО, 2010, задача 6) Пусть дан неравнобедренный остроугольный треугольник
АВС. Пусть O, I, H – центр описанной окружности, инцентр, ортоцентр треугольника
АВС соответственно. Докажите, что
a.
OIH>90;
b.
OIH<135.
13.
(МЖО, 2011, задача 6) Диагонали вписанного четырехугольника ABCD
пересекаются в точке К; пусть точки M и N – середины диагоналей AC и BD
соответственно. Пусть вписанные окружности треугольников ADM и BCM пересекаются в
точках M и L. Докажите, что точки K, L, M и N лежат на одной окружности (все указанные
точки различны).
Зачет.
1.Решить уравнения в целых числах, применяя не менее трёх разных способов.
а) 27х – 40y = 1;
б) 54x + 37y = 7;
в) 107x + 84y =1;
г) 13x – 15y =7;
д) 81x + 52y = 5;
e) 24x – 56y = 72.
2.На какое наименьшее число надо умножить 7, чтобы произведение оканчивалось на 123.
3.Кусок проволоки длиной 102 см нужно разрезать на части длиной 15 см и 12 см, так
чтобы была использована вся проволока. Как это сделать?
4.Доказать, что уравнения не имеют целочисленных решений:
а) y2 = 5x2 + 6;
б) x3 = 2 + 3y2
xy yz zx
5. Решить в целых числах уравнение
 
 3.
z
x
y
6. Положительные числа x и y меньше единицы. Докажите, что выполняется неравенство
𝑥
𝑦
+ 1+𝑥 < 1
1+𝑦
7. Докажите, что если произведение двух положительных чисел больше их суммы, то
сумма больше 4.
8.Докажите, что если a, b, c—положительные числа и ab + bc + ca > a+ b + c, то
b + c > 3.
a+
9. Найдите формулу n-го члена последовательностей, заданных условиями (n ≥ 0):
a) 𝑎0 = 0, 𝑎1 = 1, 𝑎𝑛+2 = 5𝑎𝑛+1 − 6𝑎𝑛 ;
б) 𝑎0 = 0, 𝑎1 = 1, 𝑎𝑛+2 = 3𝑎𝑛+1 − 2𝑎𝑛 ;
в) 𝑎0 = 0, 𝑎1 = 1, 𝑎𝑛+2 = 𝑎𝑛+1 + 𝑎𝑛 ;
10.Последовательность аn удовлетворяет при любом натуральном n соотношению 𝑎𝑛+2 =
𝑎𝑛+1 +1
. Найдите 𝑎1998 , если 𝑎19 = 19, 𝑎97 = 97.
𝑎
𝑛
Материалы занятий группы English-B .
Руководители А. Теслер и Ю. Петрова.
Серия 17 июля 2013
0. Кузнечик стоит на кольце, состоящем из L кирпичей. Он умеет прыгать на 6
кирпичей в любом направлении. Сумеет ли он побывать на всех кирпичах?
Рассмотрите случаи:
a) L=12; 15; 16; 23.
b) L=393; 642; 857.
c) При каких L кузнечик сможет побывать на всех кирпичах, а при каких — нет?
1. Можно ли разрезать прямоугольник 29×11 на прямоугольники 2×3 и 5×4?
2. Найдите все пары натуральных m и n, для которых:
a) 𝑚²– 𝑛² = 5; 𝑏) 𝑚²– 𝑛² = 6; 𝑐) 𝑚²– 𝑛² = 15; 𝑑) 𝑚²– 𝑛² = 222.
3. Кузнечик стоит на бесконечной в обе стороны клетчатой полоске в клетке с
номером 0. Он умеет прыгать на 3 и на 5 клеток в любом направлении.
a) Сумеет ли он достичь клетки с номером 79?
b) Верно ли, что он сумеет достичь любой клетки?
4. Лягушка, стартующая из той же клетки, умеет прыгать на 15 и на 24 клетки.
а) Сумеет ли она допрыгать до клетки 19?
b) До клетки 111?
c) Найдите все клетки, до которых она может допрыгать.
5. a) Докажите, что монетами в 6 и 19 евро можно набрать любую целую сумму,
которая больше 100 евро.
b) Пусть у покупателя и у продавца есть неограниченное количество монет в 6 и 19
евро. Докажите, что покупатель может купить товар с любой целой ценой.
6. На окраску куба со стороной 3 см использовано 4 г краски. Сколько краски
потребуется на окраску куба со стороной 7 см?
7. Треугольный материк разбит на четыре треугольных страны. Могут ли все страны
граничить друг с другом?
8. На доске написаны несколько целых чисел. Может ли случиться, что их сумма
равна 2013 и произведение — тоже 2013?
9. Можно ли на доску 5 х 5 поставить 3 коня так, чтобы они били все незанятые ими
клетки?
10. Сколько существует трехзначных чисел, у которых одна из цифр равна сумме двух
других?
Серия 18 июля 2013
Арифметика остатков
Говорят, что 𝑎 ≡ 𝑏 (𝑚𝑜𝑑 𝑚), если 𝑎 − 𝑏 кратно 𝑚.
1. Доказать, что если 𝑎 ≡ 𝑏 (𝑚𝑜𝑑 𝑚), 𝑐 ≡ 𝑑 (𝑚𝑜𝑑 𝑚), то
𝑎) (𝑎 + 𝑐) ≡ (𝑏 + 𝑑) (𝑚𝑜𝑑 𝑚); 𝑏) 𝑎𝑐 ≡ 𝑏𝑑 (𝑚𝑜𝑑 𝑚).
2. Доказать, что число 13·16·19+14·17·20 делится на 3.
3. Найти остаток от деления числа 833+1533+2833 на 7.
4. a) Найдите без калькулятора какие-нибудь числа 𝑎 и b, для которых 36𝑎 + 65𝑏
кратно 17. b) Найдите хотя бы три разных ответа к задаче (a).
Взаимно простые числа
Два числа называются взаимно простыми, если они не имеют общих делителей,
больших 1.
Свойство взаимно простых чисел: если a и b взаимно просты, ac кратно b, то c кратно
b.
5. Докажите, что если числа 𝑎 и b взаимно просты, то числа 𝑎 + 𝑏 и b тоже взаимно
просты.
6. a) Какой цифрой оканчивается число x, если последняя цифра числа 7x равна 2?
b) Найдите последнюю цифру числа y, если 6y оканчивается на 4.
Линейное представление единицы
Следующие задачи (17-20) лучше решать последовательно.
7. Рассмотрим числа 0, 22, 22·2, 22·3, 22·4, ..., 22·56. Докажите, что их остатки от
деления на 57 не совпадают (т.е. все разные).
8. Докажите, что при каком-то 𝑥
22𝑥 ≡ 1 (𝑚𝑜𝑑 57).
9. Докажите, что найдутся целые 𝑎 и b , при которых 22𝑎 + 57𝑏 ≡ 1.
10. Докажите, что найдутся целые c и d, при которых 22𝑐 + 57𝑑 ≡ 1607.
Всё то-же самое верно, если взять вместо 22 и 57 любые взаимно простые числа (и
доказывается так же).
Разное
11. Что больше: сумма всех чётных чисел от 1 до 2013 или сумма всех нечётных чисел
от 1 до 2013? На сколько? Решите задачу, не считая этих сумм.
12. Нарисуйте комнату, которую нельзя осветить одной лампочкой, но можно —
двумя. (Лампочка светит сколь угодно далеко.)
13. Существует ли точный квадрат, запись которого состоит из двоек, троек и
восьмёрок?
Серия 20 июля 2013
1. В гостинице есть комнаты от 1 до 99. Детектив ищет комнату, в которой
скрывается преступник. Он договорился с Центром, что умножит номер комнаты
на 17 и сообщит две последние цифры результата. Докажите, что этой информации
хватит Центру, чтобы узнать номер комнаты.
2. Сколько из чисел от единицы до миллиона взаимно просты: a) с числом 1000; b) с
числом 35?
3. a) Два велосипедиста любят гоняться друг за другом. Они ездят по кругу длиной
400 м, причём их скорости равны 16 м/с и 25 м/с. Они стартовали одновременно из
одной точки. Когда они впервые встретятся на старте? b) Теперь к ним
присоединился приятель, скорость которого 10 м/с. Через какое время все трое
встретятся на старте?
4. a) Альба вырезает из доски 8×8 прямоугольники, состоящие из 9 клеток. Какое
максимальное количество прямоугольников может у неё получиться? b) Другая
Альба вырезает прямоугольники площадью 12 клеток. Сколько прямоугольников
она сумеет вырезать?
5. Два посёлка A и B стоят недалеко от железной дороги. Где надо построить
станцию, чтобы суммарное расстояние от посёлков до станции было как можно
меньше?
Игры
6. Двое играют в игру. Каждый по очереди берёт из кучки конфет одну или две
штуки. Проигрывает тот, кто съел последнюю конфету (это невежливо). Кто может
обеспечить себе победу (первый или второй), если в кучке: a) 5 конфет; б) 10
конфет; в) 30 конфет?
7. Двое играют в игру: по очереди ставят на доску 9×9 по королю. Проигрывает тот,
после чьего хода какие-то короли бьют друг друга. Придумайте для одного из
игроков (первого или второго), как он должен играть, чтобы точно выиграть.
8. На доске написано число 0. За один ход можно увеличить или уменьшить его на 1
или на 3. Алекс и Боб по очереди делают это (Начинает Алекс). Алекс мечтает,
чтобы в один прекрасный момент число стало равно 10, а Боб постоянно ему
мешает. Сумеет ли Алекс добиться цели?
Серия 21 июля 2013.
Игры
1. В левом нижнем углу шахматной доски стоит хромая ладья, которая умеет ходить
только вправо и вверх. Двое игроков по очереди двигают её; побеждает тот, кто
приведёт ладью в правый верхний угол. Кто победит при правильной игре?
2. Двое по очереди ломают шоколадку 6×8. За ход разрешается сделать один
прямолинейный разлом. Тот, кто не может сделать хода, проигрывает.
3. Дано число 60. За один ход можно уменьшить его, но не более чем вдвое (число
должно оставаться целым). Выигрывает тот, кто получит 1.
4. Двое играют в игру: по очереди кладут на круглый стол одинаковые монеты (по
одной). Монеты нельзя сдвигать, они не могут накладываться друг на друга.
Проигрывает тот, кому некуда положить монету. (У обоих игроков есть точные
измерительные инструменты.)
5. a) Двое по очереди ставят в клетки шахматной доски ладьи, не бьющие друг друга.
Проигрывает тот, кто не может этого сделать.
b) То же для слонов.
6. Имеется две кучки камней: в одной 30 камней, в другой 20. За ход можно взять
любое число камней из любой кучки. Проигрывает тот, кому нечего брать.
7. Докажите, что в игре в крестики-нолики на бесконечной доске крестики могут
добиться хотя бы ничьей.
Серия 22 июля 2013.
Фибоначчи, золотое сечение и так далее
1. Задача о прыгуне. Дана полоска из n клеток. Прыгун стоит на первой её клетке
и умеет прыгать на одну или две клетки вправо. Сколько способов добраться до
последней клетки? a) n=4; b) n=5; c) n=6; d) n=10.
2. Биссектриса делит треугольник. Каковы углы треугольника, если
биссектриса делит его на два равнобедренных треугольника? Найдите все
варианты.
3. Золотое сечение. Найдите число, которое при возведении в квадрат
увеличивается на 1. Сколько таких чисел существует? (Обозначим большее из
них через τ, «тау».)
4. Прямоугольник. От прямоугольника отрезали квадрат, и получился
прямоугольник, подобный исходному (то есть с тем же соотношением сторон).
Найдите это соотношение.
5. Возведение в степень. а) Докажите, что τ3=2τ+1, а τ4=3τ+2. б) Запишите
подобным образом τ10.
6. Стороны треугольников. Найдите соотношение сторон каждого треугольника
из задачи 2.
Серия 23 июля 2013.
Ещё о числах Фибоначчи
1. У Димы есть квадратик 1×1. Каждый раз к большей стороне имеющейся у него
фигуры Дима пририсовывает квадрат, сторона которого равна этой стороне.
После 15 таких пририсовываний получился прямоугольник. Найдите его
стороны.
2. Могут ли два числа Фибоначчи подряд делиться на 37?
3. Пронумеруем числа Фибоначчи: f1=f2=1, f3=2 и т.д.
а) Чётно или нечётно сотое число Фибоначчи? б) Какой остаток оно даёт при
делении на 11?
4. Чему равен остаток от деления f100 на f99?
5. Докажите, что если какое-то из чисел Фибоначчи делится на 7, то оно делится и
на 3.
6. Докажите, что f n+1 ≤ 2n.
7. Сколькими способами можно уложить 11 домино 1×2 в прямоугольник 2×11?
8. В здании n этажей, каждый из них можно покрасить в белый или чёрный цвет.
При этом красить два этажа подряд в чёрный цвет нельзя. Сколько способов
покрасить здание?
Разное (для тех, кому надоели числа Фибоначчи))
9. Свежее сено содержит 99% воды, а подсушенное — 95%. Во сколько раз
уменьшается масса сена при сушке?
10. Какую градусную меру может иметь средний по величине угол треугольника?
11. В квадрате со стороной 5 см размещено 126 точек. Доказать, что среди них
существует 6 точек, которые лежат в круге радиуса 1 см.
Серия 24 июля 2013.
Максимизация и минимизация
1. На какое минимальное число шестиугольников можно разрезать квадрат?
2. На какое минимальное число треугольников можно разрезать десятиугольник?
3. Сумма двух различных натуральных чисел равна 100. Каково максимальное
значение их НОД?
4. Поставьте на доске 8×8 как можно меньше слонов так, чтобы они били всю
доску. Считается, что слон бьёт клетку, на которой стоит.
5. Поставьте на доске 8×8 максимально возможное количество слонов так, чтобы
они не били друг друга.
6. Разрежьте прямоугольник 4×5 на как можно большее число попарно неравных
прямоугольников (т.е. все прямоугольники должны быть различны).
7. Используя цифры от 0 до 9 по одному разу, составьте из них несколько чисел
так, чтобы сумма этих чисел была как можно ближе к тысяче.
Серия 25 июля 2013.
Геометрия
1. Найдите отношение высоты равностороннего треугольника к его стороне.
2. Диагонали разбивают трапецию на четыре треугольника. Доказать, что
треугольники, прилежащие к боковым сторонам, имеют равную площадь.
3. Сторона правильного пятиугольника равна 1. а) Найдите его диагональ. б)
Найдите отрезки, на которые диагональ делится другими диагоналями.
4. Боковую сторону трапеции разделили точками K и L на три равных части и
через эти точки провели прямые, параллельные основаниям. В результате
трапеция оказалась разрезана на три части. Докажите, что площадь средней
части равна 1/3 площади трапеции.
5. Прямоугольная (но не квадратная) картина заключена в раму, ширина рамы
всюду одинакова. Может ли внешний контур рамы быть подобен внутреннему
контуру?
6. Докажите, что любой остроугольный треугольник можно разрезать на три
равнобедренных.
7. Площадь треугольника ABC равна 12. На стороне AB отмечена точка K, для
которой AK=2KB; M — середина стороны AC. Найдите площадь
четырёхугольника KBCM.
Серия 29 июля 2013.
Математический бой
1. Книга состоит из 30 рассказов объёмом 1, 2, ..., 30 страниц. Рассказы печатаются с
первой страницы, каждый рассказ начинается с новой страницы. Какое наибольшее
количество рассказов может начинаться с нечётной страницы?
2. Даны четыре попарно различных положительных числа a, b, c и d. Каждую минуту
эти числа одновременно заменяются на a+b+c–d, a+c+d–b, a+b+d–c, и b+c+d–a.
Обязательно ли на доске через некоторое время появится отрицательное число?
3. Восемь хоккейных команд провели чемпионат: каждые две команды сыграли ровно
один матч. За победу давалось 2 очка, за ничью – 1 очко, за поражение – 0 очков.
Оказалось, что ровно семь команд поделили второе место. Сколько очков могла
набрать команда-победительница?
4. Что больше: сумма пятых степеней натуральных чисел от 1 до 10000 или сумма
десятых степеней натуральных чисел от 1 до 100?
5. ЭВМ по вставленным в нее карточкам с ненулевыми числами a и b выводит
карточку с числом 1-a/b и возвращает обе вставленные карточки. Как, имея
карточки с числами 1 и 10, получить карточку с числом 1000?
6. Фигура «сфинкс» состоит из 6 правильных треугольников со стороной 1 (см. рис).
Можно ли правильный треугольник со стороной 30 разрезать на сфинксов?
(Фигурки можно поворачивать и переворачивать.)
7. На Васиной чаше двухчашечных весов лежат гири весом 1 г, 3 г, ..., 2001 г, а на
Петиной чаше - 2 г, 4 г, ..., 2000 г. Первым ходит Вася - он убирает в некотором
порядке по одной гире со своей чаши до тех пор, пока она не станет легче Петиной.
Потом Петя убирает по одной гире со своей чаши до тех пор, пока она не станет
легче Васиной. Затем опять ходит Вася, потом Петя, и так далее. Выигрывает тот,
кто первым сможет убрать все гири со своей чаши. Кто выигрывает при
правильной игре?
8. Каждая клетка тетрадного листа закрашена в один из семи цветов. Возможно ли,
что любом L-тетрамино все клетки разного цвета? (L-тетрамино - это фигурка из
четырёх клеток в форме буквы L или Г, возможно, повёрнутая.)
Зачёт.
Теория чисел (до 10:30)
1. Найти последнюю цифру положительного числа, если его остаток от деления на 20
равен 17.
2. Известно, что 19𝑘 ≡ 2 + 4𝑘 (𝑚𝑜𝑑 8). Каков остаток от деления k на 8?
3. Посчитать остаток от деления: a) 15·17+25·27+35·37 на 6; b) 175 на 11.
4. Найти остаток от деления 4343+4444 на 7.
5. У покупателя есть много монет в a евро, а у продавца — много монет в b евро.
Может ли покупатель купить товар стоимостью в 1 евро? Рассмотрите случаи: a)
𝑎 = 52, 𝑏 = 83; b) 𝑎 = 55, 𝑏 = 143; c) 𝑎 = 410, 𝑏 = 189. При необходимости вы
можете сослаться на общее утверждение, доказанное нами (сформулируйте его).
6. Решите уравнение 𝑚²– 𝑛² = 315, где 𝑚 и 𝑛 — натуральные числа.
7. Сколько чисел, меньших числа 187, взаимно просты с ним?
Фибоначчи и тау (до 11:30)
1. Какие комбинаторные задачи, ответом в которых являются числа Фибоначчи, вы
знаете? (Комбинаторные задачи — это задачи вида «Сколько способов что-то
сделать?»)
2. Какие геометрические задачи, ответом в которых является «золотое сечение», вы
помните?
3. Найдите остаток от деления 30-го числа Фибоначчи на 9.
4. Верно ли, что 1000-е и 1002-е числа Фибоначчи взаимно просты?
5. Дом из n этажей красят в два цвета (чёрный и белый), при этом запрещено делать
три подряд идущих этажа чёрными. Сколько способов покрасить такой дом?
Найдите ответ при n=10.
(Подсказка: искомая последовательность известна под названием «числа
Трибоначчи»).
6. Докажите, что 1/𝜏 = 𝜏-1, а 1/𝜏² = 2-𝜏.
Игры (до 12:30)
1. Двое по очереди ставят на доску 8×8 белых коней так, чтобы они не били друг
друга. Проигрывает тот, кто не может сделать ход. У кого из игроков есть
выигрышная стратегия и какая? Придумайте несколько различных стратегий, если
можете.
2. На столе лежит 10000 яблок. Двое по очереди съедают часть яблок, но каждым
ходом можно съесть не более 90% всех имеющихся на столе яблок. Выигрывает
тот, после чьего хода останется одно яблоко. Кто из игроков может выиграть при
любых действиях соперника?
3. Двое вожатых играют в игру. У них есть ведро с 20 конфетами. За один ход можно
положить в ведро от 1 до 10 конфет. Выигрывает тот, после хода которого в ведре
окажется хотя бы 100 конфет. У кого из них есть выигрышная стратегия и какова
она?
4. Двое по очереди проводят в правильном 20-угольнике диагонали. Диагонали могут
иметь общий конец, но не могут пересекаться во внутренних точках. Тот, кто не
может этого сделать, проиграл. Кто из игроков (первый или второй) может
выиграть независимо от игры противника? Почему?
Геометрия (до 13:30)
1. На стороне PQ треугольника PQR стоит точка A такая, что PA=3AQ, а на стороне
PR — точка B такая, что RB=3BP. Найдите площадь ∆PAB, если площадь ∆PQR
равна 24.
2. Одно из оснований трапеции вдвое меньше другого. Два отрезка, параллельных
основаниям трапеции, делят её на три трапеции с равными высотами. Во сколько
раз площадь самой большой из этих частей больше, чем площадь самой маленькой?
3. На сторонах треугольника ABC отмечены такие точки P и Q, что отрезок PQ делит
его площадь пополам. Известно, что точка P лежит на стороне AC и AP=2PC. На
какой из сторон лежит точка Q и в какой пропорции делит её? Найдите все
варианты и объясните, почему других нет.
4. Прямоугольник можно разрезать на два прямоугольника, подобных исходному.
Найдите отношение сторон этого прямоугольника.
5. Доказать, что в треугольнике с углами 36°, 72° и 72° отношение сторон равно сами
знаете чему.
Материалы группы Russian-B
Руководители М. Бондаренко и А. Дубатовка.
Серия 1, разнообразная.
0) Докажите, что при вещественных 𝑎 и 𝑏 выполняется неравенство
𝑎2 + 𝑏 2 ≥ 2𝑎𝑏.
1) Деревни Альфино и Бетино находятся а) по разные стороны, б) по одну сторону
от прямолинейной железной дороги. В каком месте дороги нужно построить станцию для
того, чтобы сумма расстояний от деревень до станции была наименьшей?
2) В прямоугольном треугольнике ABC ∠B - прямой, а ∠A=300. Через середину
гипотенузы AC провели прямую, перпендикулярную ей. Докажите, что отрезок этой
прямой, заключенный внутри
треугольника, в три раза короче стороны AB.
3) Докажите, что при положительных 𝑥 и 𝑦 выполняется неравенство
4)Вещественные числа 𝑎, 𝑏 и с таковы, что 𝑎 + 𝑏 + 𝑐 = 1. Докажите, что
𝑎2 + 𝑏 2 + 𝑐 2 ≥ 1/3
𝑥
𝑦
+ 𝑥 ≥ 2.
𝑦
.
5) Прямоугольную клетчатую доску можно разрезать на уголки из трех клеток. Докажите,
что ее можно разрезать и на полоски 1 на 3.
6) Можно ли разрезать доску 6 на 6 на полоски 1 на 4?
7) Вася и Петя играют в увлекательную игру: по очереди кладут одинаковые
пятикопеечные монеты на круглый стол. Монеты не должны налегать друг на друга или
вылезать за край стола. Проигрывает тот, кому некуда класть монету. Первым ходит Вася.
Как он должен играть, чтобы заведомо выиграть?
8) Вася и Петя нарисовали на доске а) отрезок; б) квадрат.
Они по очереди (начинает Вася) ставят в него точки. Тот, после чьего хода на доске
оказываются две точки, расстояние между которыми меньше 1, проигрывает. Кто выиграет
при правильной игре?
9) Двое игроков играют в следующую увлекательнейшую игру: в кучке лежит а)39 б)40
спичек, за ход можно взять и сжечь либо 1, либо 2 из них. Выигрывает тот, после чьего
хода спичек в кучке не останется. Кто выигрывает при правильной игре?
10) Можно ли на шести книжных полках длиной по 1 м расставить 49 книг толщиной по 6
см и 101 книгу по 3 см?
11) Стандартную колоду из 36 карт раздали поровну двум игрокам. Каждую минуту
каждый из игроков выкладывает из своих карт на стол пару карт одного достоинства
(например, пару семерок), если она у него есть. Докажите, что они закончат этот процесс
одновременно.
12) Хоккеист играет на бесконечном ледовом поле тремя шайбами. За один ход он бьет по
одной из шайб так, чтобы она пролетела между двумя другими (т.е. отрезок, по которому
летит шайба, должен пересекать отрезок, соединяющий две другие шайбы). Могут ли все
шайбы вернуться каждая на свое место после 239 таких ударов?
Серия 1.5
Продолжение разнообразностей.
1)У доски 8 х 8 вырезали две противоположные угловые клетки. Можно ли теперь доску
покрыть доминошками?
2) Вася и Петя ломают шоколадку 6 на 4 по «швам». Проигрывает тот, кто не может
сделать ход. Кто выиграет при правильной игре?
3) На доске написаны 100 натуральных чисел: от 1 до 100. Разрешается стирать числа 𝑎 и с
и писать вместо них 𝑎𝑐. Какое число может оказаться последним?
б) тот же вопрос, когда вместо 𝑎 и с пишут 𝑎𝑐 + 𝑎 + 𝑐.
4) В стране 100 городов. Некоторые пары городов соединены дорогами, причем из
каждого города выходит ровно 5 дорог. Страна разделилась на 2 республики по 50 городов
в каждой. Жители первой республики посчитали, сколько оказалось дорог между
городами их республики. Жители второй республики тоже посчитали количество дорог в
своей республике.
Докажите, что эти два числа совпали.
5) В графстве каждый джентльмен принадлежит одной из двух партий: партии любителей
мяса или партии любителей рыбы. Каждая дорога в этом графстве соединяет усадьбы
джентльменов из разных партий. Известно, тем не менее, что каждый из джентльменов
может проехать по дорогам к любому своему соратнику по партии. Докажите, что любой
джентльмен может проехать к любому другому джентльмену.
6) Дана окружность с центром 𝑂, а также точки 𝐴, 𝐵 и 𝐶 на окружности. ∠ 𝐴𝑂𝐶
называется центральным углом, опирающимся на дугу 𝐴𝐶, а ∠𝐴𝐵𝐶 - центральным углом,
опирающимся на ту же дугу. Докажите, что ∠𝐴𝑂𝐶 вдвое больше ∠ 𝐴𝐵𝐶 в случае, если 𝑂
лежит а) внутри; б) на стороне; в) вне ∠𝐴𝐵𝐶.
7) В треугольнике 𝐴𝐵𝐶 ∠𝐴𝐵𝐶 < ∠𝐵𝐴𝐶. Докажите, что 𝐴𝐶 < 𝐵𝐶.
8) Докажите, что отрезок 𝐴𝑋, соединяющий вершину 𝐴 треугольника 𝐴𝐵𝐶 с точкой на
противоположной стороне, меньше хотя бы одной из сторон 𝐴𝐵 и 𝐴𝐶.
Серия 2.Игрушечная геометрия.
-2) Докажите, что сумма степеней вершин графа равна удвоенному количеству его рёбер.
-1) а) Докажите, что четырехугольник является вписанным тогда и только тогда, когда
сумма его противоположных углов равна 1800.
б) Докажите, что четырехугольник 𝐴𝐵𝐶𝐷 является вписанным тогда и только тогда, когда
∠ 𝐴𝐵𝐷 = ∠𝐴𝐶𝐷.
0)Дан ∆ 𝐴𝐵𝐶. Докажите, что биссектриса угла 𝐴 и серединный перпендикуляр к отрезку
𝐵𝐶 пересекаются на описанной окружности исходного треугольника.
1) В треугольнике 𝐴𝐵𝐶 точки А1, В1 и С1 являются основаниями высот, опущенных из
вершин 𝐴, 𝐵 и 𝐶 соответственно. Докажите, что а) ∆𝐴𝐵1 𝐶1 подобен ∆𝐴𝐶𝐵 (именно в таком
порядке)
б) Докажите, что AA1 -- биссектриса ∠ВА1С1 .
2) (Лемма о трезубце) Точка C1 - середина дуги AB описанной окружности ∆ABC.
Докажите, что она равноудалена от центра вписанной окружности, вершин A и B, а также
от центра вневписанной окружности, касающейся стороны AB.
3) Дан квадрат ABCD. На стороне AB взята точка K, на стороне CD - точка L, а на отрезке
KL - точка M. Докажите, что вторая (отличная от M) точка пересечения описанных
окружностей треугольников AKM и MLC лежит на диагонали AC.
4) Внутри окружности дана точка M. Через неё проведены две прямые: одна пересекает
окружность в точках A и B, другая - в точках C и D. Докажите, что ∠ AOC равен
полусумме дуг AC и BD.
5)Докажите, что 𝑎4 + 𝑏 4 + 𝑐 4 ≥ 𝑎𝑏𝑐(𝑎 + 𝑏 + 𝑐) при вещественных 𝑎, 𝑏 и 𝑐.
6) Докажите, что при 𝑥, 𝑦 ≥ 1 выполняется неравенство 𝑥𝑦 + 1 ≥ 𝑥 + 𝑦 .
7) В куче 888000 спичек. Два мудреца по очереди берут спички. В свой ход можно взять из
кучи любое количество спичек, кроме того, которое было взято противником на
предыдущем ходу (брать первым ходом все спички не разрешается). Проигрывает тот, кто
не может сделать ход. Докажите, что как бы хитро ни играл первый мудрец, второй мудрец
сможет выиграть.
8) Вася и Петя играют в игру на полоске 1 на 2011. За один ход Вася (он ходит первым)
может закрасить черной краской две соседние клетки (если ни одна из них пока не
закрашена), а Петя - три подряд идущие клетки (если ни одна из них пока не закрашена).
Проигрывает тот, кто не может сделать ход. Кто выиграет при правильной игре?
9) В начале игры на доске написано число 1000. Два игрока ходят по очереди. За один ход
можно вычесть из написанного числа любой его натуральный делитель и результат
написать на доску вместо исходного числа. Тот, кто напишет ноль, проигрывает. Кто из
игроков может
обеспечить себе победу: начинающий или его противник?
10) В начале игры на доске написано число 0. Два игрока ходят по очереди. За ход игрок
прибавляет к написанному числу любое натуральное число, не превосходящее 10, и
результат записывает на доску вместо исходного числа. Выигрывает тот, кто первым
получит четырехзначное число. Кто выигрывает при правильной игре: начинающий или
его противник?
Серия 3. Повторение - мать заикания.
1) Дан треугольник и точка внутри него. Из точки опущены перпендикуляры на стороны
(они попали на сами сторона, а не на их продолжения). Оказалось, что два из трёх
образовавшихся четырёхугольников - вписанные. Докажите, что третий четырёхугольник
тоже вписанный.
2) Докажите, что самый длинный отрезок, содержащийся в треугольнике, - это его
наибольшая сторона.
3) На плоскости нарисован прямоугольник, диагонали которого равны 2, а также отмечена
некоторая точка A. Докажите, что можно выбрать три вершины прямоугольника, сумма
расстояний от которых до точки A не меньше, чем 3.
4) Докажите, что при вещественных 𝑎 и 𝑏
а) 4𝑎2 + 𝑏 2 ≥ 4𝑎𝑏;
𝑎
𝑏
б) 𝑎2 +1 + 𝑏2 +1 ≤ 1.
5) Можно ли разрезать шахматную доску без угловой клетки на полоски 1 х 3?
6) Двое играют на доске 12 х 12. Первый игрок выставляет на доску фигурки из 4 клеток в
виде буквы Т, в второй - уголки из трех клеток. Проигрывает тот, кто не может сделать ход.
Кто выигрывает при правильной игре?
7) На столе лежит куча из 2012 спичек. Двое играют в такую игру: за один ход
начинающий может взять 1, 2, 3 или 4 спички, а второй игрок - 1, 2, 3 спички или
пропустить ход. Выигрывает тот, кто берет последнюю спичку. Кто выиграет при
правильной игре?
Серия 4, содержащая сразу несколько новых идей.
1) AA1 и BB1 - высоты остроугольного треугольника ABC, O - центр описанной
окружности. Докажите, что прямые A1B1 и CO перпендикулярны.
2На стороне AC треугольника ABC отмечена точка K. Точки M и N - середины сторон AB
и BC соответственно. Известно, что KM < BM. Докажите, что KN > BN.
3) На стороне BC треугольника ABC взята точка D. Докажите, что а)
𝑆∆𝐴𝐵𝐷
𝑆∆𝐴𝐵𝐶
=
𝐵𝐷
𝐵𝐶
б) На сторонах угла с вершиной O взяли точки: A и B на одной стороне, C и D - на другой.
𝑆
𝐴𝑂∗𝑂𝐵
Докажите, что 𝑆∆𝐴𝑂𝐵 = 𝑂𝐶∗𝑂𝐷 .
∆𝐴𝑂𝐶
4)Докажите, что сумма расстояний от точки внутри правильного треугольника до его
сторон не зависит от выбора этой самой точки.
1
1
1
1
1
5) Докажите, что 1 + + ⋯ +
+ 𝑛 > √𝑛 при всех натуральных 𝑛 > 2.
√2
√𝑛−1
√
6) Докажите, что при положительных 𝑎, 𝑏 и с выполняется неравенство
𝑏𝑐
𝑎
+
𝑎𝑐
𝑏
𝑎𝑏
𝑐
+
≥𝑎+𝑏+𝑐.
7) Можно ли разбить доску 12х12 на s-тетраминошки?
8) а)Можно ли разрезать шахматную доску на фигурки, состоящие из 4 клеток в форме
буквы "т"?
б) А можно ли разрезать на такие фигурки шахматную доску 10х10?
9) Докажите, что квадрат 10х10 нельзя разрезать на тетрамино (фигурки из четырех
клеток) в виде буквы Г.
10) а) Изначально ладья стоит в левом нижнем углу шахматной доски. Двое игроков по
очереди делают ладьёй ход, при этом ходить вниз и влево запрещено. Выигрывает тот, кто
поставит ладью в правый верхний угол. Кто выигрывает при правильной игре?
б) Та же задача, только игроки ходят королём, которым нельзя ходить вниз, вправо и "вниз
и вправо по диагонали".
11) В начале игры имеется имеется куча из 100 конфет. Малыш и Карлсон ходят по
очереди. За ход можно взять из кучи 2, 3 или 5 конфет. Проигрывает тот, кто не может
сделать ход.
Первым ходит Малыш. Кто выигрывает при правильной игре?
12) В графе 12 вершин и 56 рёбер. Докажите, что этот граф связен.
13) На крайней правой клетке доски 1х40 стоит фишка. Два игрока по очереди двигают эту
фишку вправо или влево на любое число клеток, которое еще не встречалось при
выполнении предыдущих ходов. Оставлять фишку на месте нельзя. Проигрывает тот, кто
не может сделать ход. Кто выигрывает при правильной игре: начинающий или его
противник
Серия 5, теоретико-числовая.
1. Натуральное число при делении на 1981 и 1982 дает в остатке 35. Каков остаток от
деления этого числа на 14?
2. Произведение натуральных чисел x и y равно 20092010 . Докажите, что 𝑥 + 𝑦 не делится
на 2008.
3. Одно и то же натуральное число поделили с остатком на 3, на 18 и на 48. Сумма трех
полученных остатков оказалась равна 39. Докажите, что остаток, полученный при делении
на 3, равен 1.
4. При каких натуральных n число 5𝑛 + 3 делится на 2𝑛 + 5?
5. Известно, что числа 𝑝, 𝑝2 + 2𝑝 - простые. Докажите, что 𝑝2 + 10 - простое.
6. а) Докажите, что произведение любых 6 подряд идущих натуральных чисел делится на
720.
б) Докажите, что произведение любых 𝑘 подряд идущих натуральных чисел делится на 𝑘!.
Серия 6, очередной шажок вперёд после непродолжительного отдыха.
1) У каждого депутата в думе ровно 1 друг и ровно 1 враг. Докажите, что депутатов можно
разбить на 2 нейтральные палаты, внутри которых нет ни друзей, ни врагов.
2) На доске написано 10 единиц и 10 двоек. Двое играют по следующим правилам: за ход
разрешается стереть две любые цифры и, если они были одинаковыми, написать двойку, а
если разными - единицу. Если последняя оставшаяся на доске цифра - единица, то выиграл
первый игрок, если двойка, то второй. Кто выиграет?
3) а) В четырехугольнике ABCD стороны AC и BD пересекаются в точке O. Докажите, что
ABCD - вписанный тогда и только тогда, когда 𝐴𝑂 ∗ 𝑂𝐶 = 𝐵𝑂 ∗ 𝑂𝐷.
б) Продолжения отрезков AB и CD пересекаются в точке O. Докажите, что
четырехугольник ABCD - вписанный тогда и только тогда, когда 𝐴𝑂 ∗ 𝑂𝐵 = 𝐶𝑂 ∗ 𝑂𝐷.
4) Из города А ведёт 101 дорога, а из города Б - 3. Из всех остальных городов ведёт по 10
дорог. Докажите, что из А можно доехать до Б.
5) В треугольнике ABC ∠C=3∠А , 𝐴𝐵 = 2𝐵𝐶. Докажите, что ∠𝐴𝐵𝐶 = 600 .
6) В остроугольном ∆ 𝐴𝐵𝐶 проведена высота 𝐴Н. Докажите, что если 𝐻𝐵 < 𝐻𝐶, то 𝐴𝐵 <
𝐴𝐶.
7) Стороны AB и BC треугольника ABC равны 12 и 18 соответственно. Из точки на
медиане BM опущены перпендикуляры на AB и BC. Сумма их длин оказалась равной 10.
Найдите длины этих перпендикуляров.
8) В трапеции ABCD стороны AB и CD - основания. Точка O - точка пересечения
диагоналей этой трапеции. Докажите, что 𝑆∆𝐴𝑂𝐷 = 𝑆∆𝐵𝑂𝐶 .
9) Докажите, что при положительных 𝑎 и 𝑏 выполнено неравенство
𝑎3 𝑏 + 𝑏 3 𝑎 ≥ 𝑎2 𝑏 2 .
10) а) У министра финансов есть 3 монеты, причем одна из них - фальшивая и она легче
настоящих. Как министру выявить эту монету за 1 взвешивание на весах без стрелки?
б) Среди 9 монет одна - фальшивая, и она тяжелее настоящих. Как выявить эту монету за 2
взвешивания на весах без стрелки?
11) Докажите, что при вещественных 𝑎, 𝑏 и с выполнено неравенство
𝑎2 𝑏 + 𝑏 2 𝑐 + 𝑐 2 𝑎 ≥ 3𝑎𝑏𝑐.
12) Несколько клеток доски 10 х 10, не имеющие общих сторон, покрашены в красный
цвет. Докажите, что оставшиеся клетки можно покрасить в два других цвета так, чтобы
клетки, имеющие общую сторону, были окрашены в разные цвета.
13) Какое минимальное количество квадратиков 1 х 1 нужно нарисовать на плоскости так,
чтобы квадрат 25 х 25 оказался нарисованным вместе со своими внутренними линиями?
14) Какое максимальное количество квадратиков 2 х 2 можно уместить в квадрате 9 х 9?
Серия 7, инвариантно-прощальная и с неправильным номером.
1) На сторонах параллелограмма построены квадраты. Докажите, что их центры образуют
квадрат.
2) Докажите, что выпуклый 17-угольник нельзя разрезать на 14 треугольников.
3)Дан четырехугольник со сторонами 𝑎, 𝑏, 𝑐, 𝑑 (последовательно). Докажите, что его
площадь не превосходит:
𝑎𝑏+𝑐𝑑
𝑎𝑐+𝑏𝑑
а) 2 ;
б) 2 .
4) Как расставить на квадратном ринге 5 боксеров с тем, чтобы наименьшее из расстояний
между ними было как можно больше? (Боксеров следует считать точечными).
5) Есть шоколадка 2011 х 2011 долек. Яша и Саша играют в такую игру: ход состоит в том,
что один из имеющихся прямоугольных кусков шоколада разламывают на две
прямоугольные части, причем Саша одну из получившихся частей сразу съедает
(независимо от того, кто ходил). Проигрывает тот, кто не может сделать ход. Первым ходит
Яша. Кто выиграет при правильной игре?
6) По кругу расставлены пятьдесят фишек. Два игрока по очереди убирают фишки,
выбирая каждым своим ходом любые три, пока не останется всего две фишки. Если две
оставшиеся фишки в начале не стояли рядом, выигрывает первый игрок (который
начинает игру); в противном случае выигрывает второй. Кто выиграет при правильной
игре?
7) Рёбра полного графа покрашены в 2 цвета. Докажите, что один из одноцветных графов
связен.
8) По кругу стоят 6 чисел а) одна единица, а остальные - нули б) две единицы, стоящие
через 1, а остальные - нули. Можно ли, добавляя по единице двум соседним числам,
сделать все числа равными?
9) Есть куча из 1001 апельсинов. За каждый ход большой любитель апельсинов Яша
съедает один апельсин из какой-нибудь кучи, где лежит более одного апельсина, а затем
любую кучу (в которой больше одного апельсина) делит на две произвольные части.
Можно ли через несколько ходов получить лишь кучи, состоящие из трех апельсинов?
10) На доске написано несколько чисел. Разрешается стереть два числа 𝑥 и 𝑦 и написать
вместо них 𝑥 − 2 и 𝑦 + 1. Докажите, что рано или поздно на доске появится
отрицательное число.
Серия 8, поговорим о геометрии, неравенствах и шахматных досках.
1) В равнобедренном треугольнике 𝐴𝐵𝐶 𝐴𝐵 = 𝐵𝐶 точка 𝐷 – произвольная точка
основания. Докажите, что радиусы описанных окружностей треугольников 𝐵𝐴𝐷 и 𝐶𝐵𝐷
равны.
2) Точки 𝑃 и 𝑄 - середины сторон 𝐵𝐶 и 𝐴𝐷 прямоугольника 𝐴𝐵𝐶𝐷, а 𝑅 - точка пересечения
диагоналей прямоугольника PQDC. Докажите, что угол ∠𝑃𝐴𝑅 = ∠𝐵𝑅𝑃.
3) На сторонах AC и AB прямоугольного треугольника ABC с прямым углом A взяты точки
E и F соответственно так, что угол AEF равен углу ABC. Точки 𝐸′ и 𝐹 ′ - основания
перпендикуляров, опущенных на BC из точек E и F соответственно. Докажите, что 𝐸′𝐸 +
𝐸𝐹 + 𝐹𝐹 ≤ 𝐵𝐶.
4) ABCD - квадрат. На стороне CD взята точка M, а на стороне AD - точка N. Отрезки BM и
CN пересекаются в точке E, CN и AM - в точке F , BN и AM - в точке G. Докажите, что
𝑆𝐵𝐸𝐹𝐺 = 𝑆𝐶𝐸𝑀 + 𝑆𝑀𝐷𝑁𝐹 + 𝑆𝐴𝐺𝑁 .
5) Докажите, что при положительных 𝑥, 𝑦 и 𝑧
𝑥
𝑥+𝑦
𝑦
𝑧
+ 𝑦+𝑧 + 𝑥+𝑧 ≥ 1 .
6) Точки А1 , В1 и С1 лежат на сторонах 𝐵𝐶, 𝐴𝐶 и 𝐴𝐵 треугольника 𝐴𝐵𝐶 соответственно.
ОА
ОВ
ОС
Отрезки АА1 , ВВ1 и СС1 пересекаются в точке O. Докажите, что АА1 + ВВ1 + СС1 = 1.
1
1
1
7) На шахматной доске стоят 8 ладей, причем ни одна из них не бьет ни одну из других.
Хулиган Вася взял острый ножик и разрезал доску на четыре квадрата
4 х 4. Докажите, что в левый верхний квадрат 4 х 4 попало столько же ладей, сколько в
правый нижний квадрат 4х4.
8) На шахматной доске 9 на 9 имеется 40 белых клеток и 41 черная клетка. Кирилл
вырезал какие то 9 белых клеток. Докажите, что остаток доски нельзя разрезать на уголки
из трех клеток.
Зачёт 30 июля.
1) Какое наибольшее количество s-тетраминошек можно вырезать из доски
17 х 17?
2) Две окружности пересекаются в точках A и B. Некоторая прямая, проходящая через A,
пересекает первую окружность в точке C, а вторую - в точке D. Прямая, проходящая через
B, пересекает эти окружности в точках E и F в том же порядке. Докажите, что отрезки CE
и DF параллельны.
3) Две окружности с центрами O1 и O2 пересекаются в точках A и B. Прямая, проходящая
через точку A, вторично пересекает окружности в точках B1 и B2. Докажите, что
треугольник O1BO2 подобен треугольнику B1BB2.
4) Докажите, что при положительных 𝑎, 𝑏 и 𝑐 𝑎4 𝑏 2 + 𝑏 4 𝑐 2 + 𝑐 4 𝑎2 ≥ 𝑎2 𝑏 2 𝑐 2 .
5) Докажите, что для любых чисел 𝑥 и 𝑦 выполнено неравенство
𝑥 2 + 𝑥𝑦 + 𝑦 2 ≥ 0.
6) Двое игроков по очереди берут камешки из нескольких кучек (вначале в каждой кучке
лежит по 100 камешков). Первый игрок может брать своим ходом 1 или 2 камешка из
одной кучки, а второй - 1 или 3 камешка из одной кучки. Проигрывает тот, кто не может
сделать ход. Кто выигрывает при правильной игре?
7) Двое игроков по очереди берут 3, 4 или 5 камешков из кучки с 239 камешками.
Проигрывает тот, кто не может сделать ход. Кто выигрывает при правильной игре?
8) Стороны треугольника равны 3, 4 и 5. Найдите радиус его вписанной окружности.
9) Докажите, что если все степени вершин связного графа равны 4, то после удаления
любого ребра он останется связным.
10)Каждая из девочек до завтрака не более 2 раз поболтала по телефону. Докажите, что
девочек можно разбить на три группы так, чтобы в одной группе не было болтавших
между собой до завтрака девочек.
11) Глубоко в джунглях, в капище бога Инварианта висит по кругу висят 12 бутылок.
Одна из них расположена горлышком вниз, а остальные - горлышком вверх. Жрецы
считают, что если вместо нее окажется перевернутой соседняя бутылка, то это приведет к
серьезным последствиям.
а) Если враждебный чужестранец добьется этого, переворачивая по 4 бутылки подряд, то
он вызовет гром;
б) по 3 - землетрясение;
в) по 6 - конец света.
Каких бедствий можно не опасаться?
12) На плоскости нарисован правильный треугольник со стороной 1. Разрешается двигать
любую его вершину по прямой, проходящей через эту вершину и параллельной стороне,
образованной двумя другими вершинами. Можно ли при помощи таких операций
получить прямоугольный треугольник с катетами длины 1?
Игра обязательно должна присутствовать в детском
коллективе. Детский коллектив, не играющий, не будет
настоящим детским коллективом. В детском возрасте
игра это норма и ребенок должен всегда играть, даже
когда делает серьезное дело”.
А.С.Макаренко
Здесь Вашему вниманию представлены материалы математических игр, проводимых в
рамках занятий. Некоторые из них (например, «Домино») проводились одновременно в
нескольких группах. Правила игр Вы найдёте чуть ниже.
Математический бой.
1. Книга состоит из 30 рассказов объёмом 1, 2, ..., 30 страниц. Рассказы печатаются с
первой страницы, каждый рассказ начинается с новой страницы. Какое наибольшее
количество рассказов может начинаться с нечётной страницы?
2. Даны четыре попарно различных положительных числа a, b, c и d. Каждую минуту эти
числа одновременно заменяются на a+b+c–d, a+c+d–b, a+b+d–c, и b+c+d–a. Обязательно
ли на доске через некоторое время появится отрицательное число?
3.Восемь хоккейных команд провели чемпионат: каждые две команды сыграли ровно
один матч. За победу давалось 2 очка, за ничью – 1 очко, за поражение – 0 очков.
Оказалось, что ровно семь команд поделили второе место. Сколько очков могла набрать
команда-победительница?
4.Что больше: сумма пятых степеней натуральных чисел от 1 до 10000 или сумма десятых
степеней натуральных чисел от 1 до 100?
5.ЭВМ по вставленным в нее карточкам с ненулевыми числами a и b выводит карточку с
числом 1-a/b и возвращает обе вставленные карточки. Как, имея карточки с числами 1 и
10, получить карточку с числом 1000?
6.Фигура «сфинкс» состоит из 6 правильных треугольников со стороной 1 (см. рис).
Можно ли правильный треугольник со стороной 30 разрезать на сфинксов? (Фигурки
можно поворачивать и переворачивать.)
7.На Васиной чаше двухчашечных весов лежат гири весом 1 г, 3 г, ..., 2001 г, а на Петиной
чаше - 2 г, 4 г, ..., 2000 г. Первым ходит Вася - он убирает в некотором порядке по одной
гире со своей чаши до тех пор, пока она не станет легче Петиной. Потом Петя убирает по
одной гире со своей чаши до тех пор, пока она не станет легче Васиной. Затем опять ходит
Вася, потом Петя, и так далее. Выигрывает тот, кто первым сможет убрать все гири со
своей чаши. Кто выигрывает при правильной игре?
8.Каждая клетка тетрадного листа закрашена в один из семи цветов. Возможно ли, что
любом L-тетрамино все клетки разного цвета? (L-тетрамино - это фигурка из четырёх
клеток в форме буквы L или Г, возможно, повёрнутая.)
ДОМИНО.
0-0
На каждой клетке доски 1010 вначале стоит по ладье. Каждым ходом можно снять с
доски ладью, которая бьет нечетное число ладей. Какое наибольшее число ладей
можно снять? (Ладьи бьют друг друга, если они стоят на одной вертикали или
горизонтали и между ними нет других ладей).
Ответ:95.
6-6
Восстановите по сохранившимся номерам путь коня в таблице 6×6 проходящий по
всем клеткам по одному разу (то есть дозаполните ее числами от 1 до 36 так чтобы
числа отличающиеся на 1 были в клетках связанных ходом коня).
17
2
23
30
15
8
24
31
16
9
22
29
3
18
1
28
7
14
32
25
10
19
36
21
11
4
27
34
13
6
26
33
12
5
20
35
Ответ:
6-5
Саша разрезал головку сыра на 10 кусков и съел самый маленький кусок. Потом он
разрезал один из кусков на два, и съел самый маленький кусок из десяти. Потом он
снова разрезал один из кусков на два, и съел самый маленький кусок из десяти.
Какую наибольшую долю головки мог съесть Саша?
Ответ: 1/4.
6-4
Шерлок Холмс хочет добыть ключ из лабиринта, в который сам попасть не может,
однако может запустить туда маленького робота. Робот знает
команды:
Вверх (передвинуться на одну клетку вверх)
Вниз (передвинуться на одну
17
11
клетку вниз)
2
25
Влево (передвинуться на одну
клетку влево)
23
16
1
Вправо (передвинуться на одну
30
19
клетку вправо)
15
13
Команду, которую робот не
8
35
может выполнить, он просто
пропускает и переходит к следующей. У робота ограниченный объем памяти,
поэтому Холмс может написать программу, состоящую только из четырех команд.
Когда робот доходит до конца своей программы, он начинает ее заново. Когда
робот оказывается в клетке, где лежит ключ, выполнение программы немедленно
завершается.
Придумайте, как Холмсу запрограммировать робота так, чтобы он попал из клетки S в
клетку F, где лежит ключ?
Ответ: К нужному результату, как нетрудно убедиться, приводит следующая
программа:
Вправо, Вправо, Влево, Вверх.
5-5
На рисунке изображен квадрат, выложенный из 12 спичек. Его площадь равна 9
квадратным спичкам. Выложите из этих же 12 спичек фигуру площадью 4
квадратных спички. Ломать, гнуть, перекрещивать спички нельзя. Никаких
висящих спичек не должно быть. В фигуре должны использоваться все 12 спичек.
Ответ:
6-3
Найдите наименьшее натуральное число, которое получается выписыванием друг за
другом 14 различных натуральных чисел.
Ответ: 101 111 213 142 345 678 9
5-4
В акционерном обществе «Елки-палки» всего 2010 акционеров. Известно, что любые
1015 из них в совокупности обладают контрольным пакетом (то есть, не менее, чем
половиной акций). Какую наибольшую долю акций может иметь один акционер?
Ответ: 3/290 (или 30/29 %).
6-2
Молоко и сливки продаются в одинаковых бутылках. За 5 пустых бутылок в магазине
можно получить 1 полную бутылку молока, а за 10 пустых бутылок — 1 бутылку
сливок. Сережа нашел в подвале 60 пустых бутылок и понес их в магазин. Получая
при обмене полную бутылку, он выпивал молоко или сливки, а освободившуюся
бутылку использовал в последующих обменах. В конце этой деятельности у него
осталась всего одна пустая бутылка. Сколько обменов совершил Сережа?
Ответ: 11.
5-3
Если от некоторого двузначного числа отнять 2, то результат разделится нацело на 3,
а если отнять 3, то результат будет делиться на 2. Если к этому числу прибавить 4,
то результат разделится на 5, а если от него отнять 5, то будет делиться на 4. Более
того, если от этого числа отнять 5, то результат разделится на 6. И еще, если к
этому замечательному числу прибавить 7, то результат разделится на 8, а если
прибавить 8, то разделится на 7. Что же это за число?
Ответ: 41.
4-4
Нарисуйте какой-нибудь 10-угольник, который можно разрезать на 5 треугольников.
Ответ:
6-1
В четырехзначном числе каждую цифру увеличили на 1 или на 5, в результате чего
оно увеличилось в 4 раза. Каким могло быть исходное число? Найдите все
варианты.
Ответ: 1717.
5-2
Найдите наибольшее натуральное число из различных цифр, у которого в любой паре
соседних цифр одна цифра делится на другую.
Ответ: 9362841705
4-3
Два автомобиля одновременно выехали из пунктов А и В навстречу друг другу. Через
7 часов езды расстояние между ними было 136 км. Найдите расстояние между А и
В, если все расстояние один проезжает за 10 часов, а другой — за 12.
Ответ: 480.
6-0
Придумайте 100-значное натуральное число без нулевых цифр такое, что оно делится
на сумму своих цифр.
Простейший пример: число, оканчивающееся на 125 и с суммой цифр 125.Проверять
тщательно!
5-1
Двадцать четыре спички выложены так, как показано на рисунке. Уберите 6 спичек
так, чтобы образовалось 5 равных квадратов.
Ответ:
4-2
Свечи Алана и Беатрис имеют один и тот же размер. Свечи Беатрис и Клер имеют
один и тот же цвет. Свечи Клер и Даниэля разного размера. И, наконец, свечи
Даниэля и Алана разного цвета. Какая из этих свечей принадлежит Элоди
(перечислите все возможности)?
Ответ: 2я или 4я
3-3
Найдите наименьшее десятизначное число, у которого в любой паре соседних цифр
одна цифра делится на другую.
Ответ: 1010101010
5-0
Сумма квадратов 2006 некоторых натуральных чисел — точный квадрат. Найдите
наименьшее возможное значение суммы этих чисел.
Ответ: 2011
4-1
Добавьте нечетное количество спичек, разделив квадрат 4×4 (16 спичек) на четыре
области по 4 клетки каждая. Гнуть, ломать и перекрещивать спички нельзя.
Решение:
3-2
У Пети 43 монеты и он хочет разложить их по карманам так, чтобы в них было разное
количество монет. При каком наибольшем количестве карманов он это сможет
сделать?
Ответ: 9 карманов.
4-0
Найдите все простые числа, которые нельзя записать в виде суммы двух составных
чисел.
Ответ: 2, 3, 5, 7, 11.
3-1
На вечеринке в честь Нового года участвовали 43 школьника. Известно, что первая
девочка танцевала с восемью мальчиками, вторая – с девятью,…, последняя – со
всеми мальчиками. Сколько было девочек на празднике?
Ответ: 18.
2-2
Сколько всего треугольников изображено на рисунке?
Ответ: 17.
3-0
По кольцевой линии метро курсируют 24 поезда. Они идут в одном направлении с
одинаковыми скоростями и одинаковыми интервалами. Сколько поездов надо
добавить, чтобы при той же скорости уменьшить интервалы на 1/5?
Ответ: 6.
2-1
В трех ящиках лежат орехи. В первом на 6 орехов меньше, чем в двух других вместе,
а во втором — на 10 меньше, чем в первом и третьем вместе. Сколько орехов в
третьем ящике?
Ответ: 8.
2-0
Сколько существует двузначных чисел, у которых цифра десятков меньше цифры
единиц?
Ответ: 36.
1-1
Сумма четырех последовательных чисел равна 20100. Найдите эти числа.
Ответ: 5022, 5024, 5026, 5028.
1-0
У овец и кур вместе 36 голов и 100 ног. Сколько овец?
Ответ: 14.
Мини-матбой.
1)Автобус идет под гору со скоростью 30 км/ч, а в гору - со скоростью 15 км/ч. На дороге
между двумя посёлками нет горизонтальных участков, а дорога автобуса туда и обратно
занимает ровно 4 часа. Найдите расстояние между посёлками.
2)Можно ли разбить цифры от 0 до 9 на две группы так, чтобы произведение цифр в
одной группе равнялось сумме цифр в другой группе?
3)За круглым столом сидят 7 человек — рыцари и лжецы. Рыцари всегда говорят правду, а
лжецы всегда лгут. Все семеро по очереди произнесли фразу: «Оба моих соседа —
лжецы». Сколько рыцарей может сидеть за столом? Укажите все возможные варианты
ответа и докажите, что других нет.
4)На квадратном куске хлеба лежит круглый кусок колбасы (не обязательно в центре).
Доекажите, что можно одним прямолинейным разрезом поделить пополам и площадь
хлеба, и площадь колбасы.
5)На чудо-дереве растут яблоки, бананы и апельсины. При срывании фрукта одного из
видов на дереве вырастают по одному фрукту другого вида (например, вместо яблока —
апельсин и банан). Всегда ли можно добиться того, чтобы ананасов на дереве было
больше, чем остальных фруктов вместе взятых?
6)Есть 21 бочка, из которых 7 пустых, 7 полных вина, а 7 наполнены вином ровно
наполовину. Разделите их между тремя людьми так, чтобы каждому досталось поровну и
бочек, и вина.
КАРУСЕЛЬКА.
Проводилась по упрощённым правилам «Математической карусели» без исходного и
зачётного рубежей.
1. На сколько процентов треть половины больше половины девятой части?
2. Из кубика 10×10×10 выпилили угловой кубик 2×2×2. Полученную фигуру окрасили, а
потом распилили на единичные кубики. Сколько при этом получилось кубиков, у которых
ровно две окрашенные грани?
3. В ящике лежат 100 тапок одного размера: по 50 правых и левых, из них 39 белых и 61
черных. Какое наименьшее число тапок надо «вслепую» достать из ящика, чтобы среди
них при любой раскраске тапок наверняка оказалась пара (левый и правый) одноцветных?
4. Сколько существует трехзначных чисел, все цифры которых — простые числа и все
двузначные числа, которые можно получить, стерев одну из этих цифр, — тоже
простые?
5. Найдите наименьшее натуральное число, десятичная запись которого содержит все
цифры от 0 до 5, и которое делится на все эти цифры (кроме, конечно, нуля).
6. Найдите такое целое k, что 2007! делится на 2007k, но не делится на 2007k+1.
7. Найдите все пятизначные числа, у которых каждая цифра числа строго больше суммы
цифр, стоящих правее нее (в частности, четвертая цифра больше пятой).
8. Числа 2007 и 1917 разделили с остатком на одно и то же число. В первом случае в
остатке получилось 88, во втором — 99. На какое число делили?
9. Какое наименьшее число участников может быть в математическом кружке, если
известно, что девочек в нем меньше 56%, но больше 53%?
10. Пятеро крестьян собрали урожай. Первый решил, что собрал больше остальных, и
разделил между ними поровну 1/3 своего зерна. После этого второй решил поделиться с
остальными, и сделал то же самое, что и первый. В результате весь урожай разделился
поровну. Определите, сколько собрал каждый, если общий вес
11. В группе туристов меньше 100 человек. Из них 12% кировчан и 18% пермяков.
Сколько туристов в этой группе?
12. На какое наименьшее число квадратов можно разрезать прямоугольник размером 6×7?
13. В однокруговом турнире по футболу (каждый с каждым сыграл ровно одну партию)
участвовало 8 команд, которые набрали 15, 14, 13, 9, 8, 7, 4 и 3 очка. За победу
присуждалось 3 очка, за ничью – 1 очко, за поражение – 0 очков. Сколько матчей в
турнире закончилось вничью?
14. На дискотеку собрался весь класс — 22 человека. Аня танцевала с семью мальчиками,
Белла — с восемью, Вера — с девятью, и так далее. Последняя из них танцевала со всеми
мальчиками. Сколько мальчиков было на дискотеке?
15. В каждой клетке доски 4х4 стоит по фишке. Каждую фишку переложили на соседнюю
по стороне клетку. Какое наибольшее количество пустых клеток могло получиться после
такого перекладывания?
16. Среди чисел 1, 2, …, k ровно 223 числа, делящихся на 9, и ровно 250 чисел, делящихся
на 8. Чему может равняться число k (перечислите все возможности)?
17. Назовем тройку простых чисел отличной, если произведение этих чисел в пять раз
больше их суммы. Найдите все отличные тройки.
18. Из натурального числа вычли сумму его цифр, из полученного числа снова вычли
сумму его цифр и так далее. После 11 таких вычитаний впервые получился ноль. С какого
числа начали?
19. По кругу расставлены 10 красных и 15 синих фишек. Обозначим через p количество
пар соседних синих фишек. Какие значения может принимать число p?
Ещё один мини-матбой.
1)Есть три столбика разной высоты. За один ход можно отпилить от любого столбика
разность высот остальных двух. Можно ли такими ходами добиться того, что все столбики
станут одной высоты?
2)Автобус идет под гору со скоростью 30 км/ч, а в гору - со скоростью 15 км/ч. На дороге
между двумя посёлками нет горизонтальных участков, а дорога автобуса туда и обратно
занимает ровно 4 часа. Найдите расcтояние между посёлками.
3) Пусть 𝑛 — некоторое натуральное число. Известно, что числа 3𝑛– 1 и 𝑛– 10 делятся на
простое число p. Найдите, чему равно p.
4) Мальчик Вася купил книгу, которая стоит целое число копеек. Сколько стоит эта книга,
если 10 книг дороже 11 рублей, а 9 книга - дешевле 10 рублей? Не забудьте объяснить,
почему другие варианты ответа невозможны.
5) Вася со своим другом Петей ехали на кружок на трамвае, и купили у кондуктора два
последовательных билетика (номер каждого билетика - шестизначное число). Петя
заметил, что сумма цифр его билета делится на 7. Вася посмотрел на свой билетик и
закричал, что и у него сумма цифр тоже делится на 7. Могло ли такое случиться?
6) Известно, что числа 𝑝, 𝑝2 + 2𝑝 - простые. Докажите, что 𝑝2 + 10 - простое.
МАТЕМАТИЧЕСКАЯ
ДУЭЛЬ.
ВАРИАНТ 1.
1.Сколько из написанных ниже утверждений являются верными?
Если числа m и n делятся на 5, то m+n делится на 5;
Если m+n делится на 7, то m делится на 7 и n делится на 7;
Если m делится на 4, а n делится на 6, то их произведение делится на 24.
Если произведение двух чисел делится на 24, то одно из них делится на 4, а другое – на 6.
2. На одной из двух параллельных прямых выбрано 8 точек, а на другой – 6 точек.
Подсчитайте число отрезков с концами на второй прямой.
3. Когда Петя начал решать эту задачу, он заметил, что часовая и минутная стрелки
его часов образуют прямой угол. Пока он решал ее, угол все время был тупым, а в
тот момент, когда Петя закончил решение, угол снова стал прямым. Сколько времени
Петя решал эту задачу?
4. Найдите углы прямоугольного треугольника, если известно, что один его угол в 5 раз
больше другого.
5.Поезд проезжает мимо столба за 20с. Сколько времени будет он ехать мимо встречного
поезда той же длины, едущего с той же скоростью
6. Найти количество треугольников, стороны которых – различные простые числа меньше
20.
7.Найти все x ,которые при умножении на 3 и делении на 7 дают остаток 2.
8. Число 100 разделили на некоторое число, меньшее 50, с остатком 6. На какое число
могло произойти деление?
9. На соревновании по бегу на дистанцию 10 км Джонни Джоггер пробежал 9 641 м,
потом прошел 3 456 дм, наконец, прополз 12 340 мм и остановился, не в силах двигаться
дальше. Сколько сантиметров ему осталось до финиша?
10.Найдите наименьшее натуральное число n такое, что среди любых n натуральных чисел
найдутся два числа, сумма или разность которых делится на 3.
ВАРИАНТ 2.
1.В алфавите 22 согласных и 10 гласных букв. Сколько можно составить двухбуквенных
слов со следующим условием: буквы должны быть различные и в слове обе буквы либо
гласные, либо согласные.
2. Число а при делении на 20 дает остаток 2. Какие остатки возможны при делении этого
числа на 8?
3.Какой угол образуют стрелки часов в 12 часов 20 минут?
4. Найти наименьшее четырехзначное число, у которого сумма цифр в десятичной записи
больше, чем у любого меньшего числа .
5.Найдите остаток от деления числа 780780 ...780 (300 цифр) на 9.
6. Планеты А и В вращаются по круговым орбитам вокруг Солнца. Планете А требуется
12 земных лет, чтобы сделать полный оборот вокруг
Солнца, а планете В – 16 лет. В настоящий момент обе планеты
находятся на одной прямой линии с Солнцем. Через какое наименьшее
время обе планеты снова окажутся на одной прямой линии с Солнцем?
7. Какой цифрой оканчивается разность 1234…212223 - 1357…192123?
8. Найдите последнюю цифру числа: 20112012 + 20122013
9. В автомате по продаже каучуковых мячиков лежат 10 белых, 20 синих, 30 чёрных и 40
красных мячиков. В автомат можно бросить монетку и получить мячик случайного цвета.
Миша бросает монетки по одной и после каждого броска считает, сколько у него мячиков
каждого цвета. Когда все четыре числа оказались различны, он останавливается. Какое
наибольшее число монеток может понадобиться Мише?
10.Витя старше Оли на четыре года, а Серёжа младше Оли на три года. При этом Витя
вдвое старше Серёжи. Сколько лет Оле?
ВАРИАНТ 3.
1.Сколько из написанных ниже утверждений являются верными?
Если число n делится на 28, то оно делится на 7;
Если число n делится на 3 и на 4, то оно делится на 12;
Если число n делится на 4 и делится на 6, то оно делится на 24;
Если число n делится на 30 и делится на 35, то оно делится на 210.
2. Поставьте в числе 341163 вместо смайлика цифру так, чтобы полученное число
делилось на 99.
3. Из проволоки спаян каркасный кубик с ребром 3см. Какой максимальной длины кусок
проволоки можно вырезать из этого каркаса?
4. Катя, Лена, Маша и Нина участвовали в концерте. Каждую песню пели три девочки.
Катя спела 8 песен – больше всех, Нина спела 5 песен – меньше всех. Сколько песен было
спето?
5.Алик, Боря и Вася собирали грибы. Боря собрал грибов на 20% больше, чем Алик, но
на 20% меньше, чем Вася. На сколько процентов больше, чем Алик, собрал грибов Вася?
6. Найдите наименьшее натуральное число, дающее при делении на 2, 3, 5, 7 остатки 1, 2,
4, 6 соответственно.
7.Число а при делении на 7 дает остаток 2, а при делении на 3 – остаток 1. С каким
остатком это число делится на 21?
8. Найдите p, если: p, 2 p + 1, 4 p + 1 – простые числа.
9. Произведение, каких четырёх натуральных последовательных чисел является числом
3024?
10. В школе прошли три олимпиады. Оказалось, что в каждой из них участвовало по 50
человек. Причем, 60 человек приходило только на одну олимпиаду, а 30 человек - ровно
на две. Сколько человек приняло участие во всех трех олимпиадах?
ВАРИАНТ 4.
1. Маша и Саша вышли одновременно навстречу друг другу. Каждый из них идёт с
постоянной скоростью и, дойдя до конца дороги, поворачивает обратно. Первый раз они
встретились через две с половиной минуты после начала движения. Когда они встретятся
во второй раз?
2. Сколько натуральных делителей у числа 7007?
3. Какие цифры надо поставить вместо звездочек, чтобы число 454** делилось
одновременно на 2, 7 и 9? Найдите все решения .
4. Цифры некоторого четырехзначного числа являются последовательными цифрами
(слева направо). Если поменять местами первые две цифры, то получится точный квадрат.
Найти число .
5. Найдите общую формулу записи числа, которое как при делении на 6, так и при
делении на 8 дает в остатке 5.
6. Какое двузначное число увеличивается в 4,5 раза, если его прочитать справа на лево.
7. Сколько существует двузначных чисел, делящихся на 5 и на 9?
8. Робинзон Крузо каждый второй день пополняет запасы питьевой воды из источника,
каждый третий день собирает фрукты и каждый пятый день ходит на охоту. Сегодня у
Робинзона тяжёлый день: он должен делать все эти три дела. Когда у Робинзона будет
следующий тяжёлый день?
9. На кольцевой трассе длиной 100 км стоят километровые столбы с номерами от 0 до 99.
Два автомобиля, стартовав от столба с номером 83 в разные стороны, первый раз
встретились у столба 8. После этого более медленный автомобиль поехал в обратную
сторону, а быстрый продолжил движение. У какого столба автомобили встретятся в
следующий раз? Скорость каждого из автомобилей постоянна.
10. Числа 100 и 90 разделили на одно и то же число. В первом случае получили в остатке 4,
во втором – 18. На какое число делили?
ВАРИАНТ 5.
1. Площадь прямоугольного катка ледового катка равна 1000 м2. Какая площадь будет у
точной копии этого катка, уменьшенной в 10 раз?
2. В графе восемь вершин. Из каждой вершины выходит три ребра. Сколько ребер в
графе?
3.У Васи есть пять палочек длины 1, 2, 3, 4, 5 сантиметров. Он выбирает три из них и
составляет треугольник. Сколько различных треугольников он может составить?
4. Найти наибольшее десятизначное число, кратное 7, все цифры в десятичной записи
которого различны .
5. Дан бесконечный ряд чисел: 2, 6, 12, 20, 30, 42, ... . Укажите закономерность и найдите
число, стоящее на 2013 – ем месте.
6.Какой должна быть цифра a в записи числа 78017801 ...780 a (200 цифр), чтобы это
число делилось на 16?
7. Сколько существует четырёхзначных чисел, которые при зачёркивании первой цифры
уменьшалось в 9 раз.
8. Квадрат 300 × 300 разбит красными линиями на «вертикальные» прямоугольники 3 × 2,
а синими линиями — на «горизонтальные» прямоугольники 2 × 3. Если провести разрезы
по всем линиям, сколько получится отдельных квадратиков 1 × 1?
9. Напишите наименьшее чётное восьмизначное число с суммои� цифр 52.
10.Какой цифрой оканчивается произведение всех простых чисел, которые меньше 100?
ВАРИАНТ 6.
1.Один покупатель купил 20% имевшегося куска полотна, второй покупатель 30%
остатка, а третий – 40% нового остатка. Сколько (в процентах) полотна осталось
непроданным?
2.Из класса в 20 человек выбирают группу тех, кто пойдёт в театр. Число человек в такой
группе может быть любым от 0 до 20 человек. Каким числом способов можно сделать
такой выбор?
3.Три команды A, B, C провели друг с другом несколько тренировочных матчей.
Известно, что команда A участвовала в 6 матчах, команда B – в 7, С – в 11. Сколько
матчей сыграли друг с другом команды А и С?
4.Вычислить сумму 1 
1 1 1 1 1
1
.
     ... 
2 4 8 16 32
128
5. Найдите последнюю цифру числа 350.
6. Найдите все такие a, что для любого b существует ровно одно c, для которого ab 3  c 2 .
7. Когда сумму цифр двузначного числа сложили с ее квадратом, то получили данное
число. Найдите это число. Необходимо указать все возможные варианты.
8. Четыре царевны загадали по двузначному числу, а Иван загадал четырёхзначное число.
После того, как они написали свои числа в ряд в каком-то порядке, получилось число
132040530321. Наидите число Ивана.
9. Найдите двухзначное число, которое вдвое больше произведения своих цифр
10.Автомат отрезает от помещенного в него прямоугольника квадрат со стороной, равной
меньшей из сторон прямоугольника. Применяя несколько раз подряд этот автомат к
имеющемуся прямоугольнику, Вася в конце концов разрезал его на 3 больших квадрата, 2
квадрата поменьше и 6 маленьких квадратов со стороной 1 см. Укажите размеры
исходного прямоугольника.
ВАРИАНТ 7.
1.Прямой забор стоит на 10 больших столбах, между каждыми двумя большими – 5
маленьких столбов. Расстояние между любыми двумя соседними столбами – 1,5 м. Какова
длина всего забора?
2. Семь гномов построились по росту, чтобы Белоснежка раздала им 707 грибов. Сначала
она дает сколько-то грибов самому маленькому. Каждый следующий получает на 1 гриб
больше, чем предыдущий. Сколько грибов получит самый большой?
3.Два велосипедиста движутся по круговому пути в одном направлении. Первый
проезжает весь круговой путь за 6 мин, а второй – за 4 мин. Второй начинает движение на
3 мин позднее первого из того же пункта, что и первый. Когда второй велосипедист
догонит первого?
4.По шпалам ходят так: либо на следующую шпалу, либо через одну шпалу. Сколькими
способами можно добраться с первой шпалы на тринадцатую?
5. На доске написано число 76. Каждую минуту число стирают с доски и на его место
записывают произведение его цифр, увеличенное на 12. Какое число окажется на доске
через час?
6. Сколькими способами можно разменять 2007 рублей 1- и 5- рублёвыми монетами.
7. Сколько делителей у числа 7007?
8. В магазине было 6 ящиков яблок, массы которых равны соответственно 15, 16, 18, 19,
20 и 31 кг. Две фирмы приобрели 5 ящиков, причем одна из них взяла в два раза больше
яблок (по массе), чем другая. Какой ящик остался в магазине?
9.Укажите точную дату и время середины 2013 года.
10.У мамы и папы в шкафу 40 книг, каждую из которых кто-то из них читал. Папа прочел
29 из этих книг, а мама — 31. Сколько книг прочитали и мама, и папа?
ВАРИАНТ 8.
1.Маша и Саша, поссорившись, пошли с равными скоростями в противоположные
стороны. Через 3 минуты Саша решил помириться и, развернувшись, стал догонять Машу,
увеличив скорость в три раза. Сколько пройдёт минут, прежде чем он догонит Машу?
2. На одной из двух параллельных прямых выбрано 8 точек, а на другой – 6 точек. Пусть
отрезок АВ соединяет третью точку на первой прямой с четвёртой точкой на второй.
Сколько отрезков, соединяющих две точки, лежащие на разных прямых, пересечёт
отрезок АВ внутри полосы?
3. Какое наименьшее число сомножителей нужно вычеркнуть в произведении 1·2·3·…·99,
чтобы произведение оставшихся сомножителей оканчивалось цифрой 2?
4.Найдите наибольшее возможное отношение трехзначного числа abc к числу ac  bc .
5.Имеется один большой ящик. В нем лежат еще два меньших ящиках. В некоторых из
них лежат еще по два ящика и т. д. Известно, что всего имеется 13 пустых ящиков.
Найдите число заполненных.
6. Мама в 4 раза старше своей дочери. Через 20 лет она будет вдвое старше ее. Сколько
лет дочери сейчас?
7. Решить в целых числах уравнение:
13x  5 y  1
8. Из доски 8×8 Миша вырезал по клеткам прямоугольник, а Ксюша разрезала его (тоже
по клеткам) на 9 фигурок, и все фигурки оказались разной площади. Какова минимально
возможная площадь Мишиного прямоугольника?
9.На прямой 30 точек, между любыми двумя соседними равно 2 см. Какое расстояние
между двумя крайними?
10.Найдите наименьшее составное число, которое не делится ни на одно из натуральных
чисел от двух до десяти.
Вариант 9.
1. Найдите значение выражения 20102 – 20092 + 20082 –…+ 22 – 12
2. Сколько существует натуральных чисел, у которых самый большой делитель (не считая
самого числа) равен 47?
3. В классе больше 30 человек, но меньше 40. Любой мальчик дружит с тремя девочками,
а любая девочка – с пятью мальчиками. Сколько человек в классе?
4.Какое наименьшее число переливаний необходимо для того, чтобы с помощью 7- и 11литровых сосудов и крана с водой отмерить 2л?
5.Поезд проезжает мимо столба за 30с. Сколько времени будет он ехать мимо встречного
поезда той же длины, едущего вдвое быстрее?
6. Какое наибольшее количество клеток на шахматной доске 8 на 8 может пересечь
прямая линия?
7. Вася купил в киоске лапшу «Доширак» и «Роллтон», всего десять пачек. Если бы все
десять пачек были «Роллтонами», то он заплатил бы на девять рублей меньше, а если бы
все пачки были «Дошираками», то на двадцать один рубль больше. На сколько рублей
пачка «Доширака» дороже, чем пачка «Роллтона»?
8. Найдите остаток от деления числа 2004·2005·2006 + 20073 на 7.
9.Книга в переплёте стоит 5 рублей 50 копеек. Сколько стоит переплёт, если книга дороже
переплёта на 5 рублей?
10.Сколько среди тысячи первых натуральных чисел таких, в записи которых встречаются
три одинаковые цифры?
Вариант 10.
1. Найдите наименьший угол прямоугольного треугольника, у которого один угол в 3 раза
больше другого.
2. Есть 7 болтов попарно различных цветов и 7 гаек таких же цветов. Сколькими
способами можно навинтить гайку на болт так, чтобы их цвета не совпали?
3. Произведение числа 21 на некоторое четырехзначное число – точный куб. Найдите это
четырехзначное число.
4. Какой цифрой оканчивается разность 1234…212223 - 1357…192123?
5. На плоскости проведено 2000 прямых общего положения. Сколько точек пересечения у
этих прямых?
6. Найти наибольшее трёхзначное число, которое при делении на 43 даёт остаток, равный
частному.
7.Сколько существует двузначных чисел, делящихся на 5 или на 9?
8. В классе 28 человек. Преподаватель физкультуры выяснил, что девочек, умеющих
плавать, в четыре раза больше, чем мальчиков, не умеющих плавать, а мальчиков,
умеющих плавать, в пять раз больше, чем девочек, не умеющих плавать. Сколько в классе
девочек?
9. К числу 10 припишите справа и слева по одной цифре так, чтобы получилось число,
кратное 72.
10.Говядина без костей стоит 300 рублей за килограмм, говядина с костями — 260 рублей
за килограмм, а кости без говядины — 50 рублей за килограмм. Сколько граммов костей в
килограмме говядины с костями?
Вариант 11.
1.Из колоды в 36 карт выбирается 3 карты. Сколькими способами это можно сделать?
2. На одной из двух параллельных прямых выбрано 8 точек, а на другой – 6 точек.
Сколько можно построить треугольников с вершинами на первой прямой и основаниями
на второй?
3. Четыре команды A, B, C и D провели друг с другом несколько тренировочных матчей.
Известно, что команда A участвовала в 6 матчах, команда B – в 5, С – в 7, D – в 10.
Сколько всего состоялось матчей?
4. Найти все натуральные n, при которых 19n  17 – целое число.
7 n  11
5.Собрались рыбаки. Один рыбак сказал так: “Вот был судак, так судак! Его длина 40 см и
еще 60% этого.” Второй рыбак сказал: “Вот был сом, так сом! Длиною 40 см и еще 60%
своей длины.” На сколько одна рыба длинее другой ?
6. В классе девочек вдвое больше, чем мальчиков. На Новый год каждая девочка вручила
открытку каждому мальчику, каждый мальчик вручил открытку каждой девочке. Всего в
обмене участвовало 324 открытки. Сколько детей в классе?
7. Решить в целых числах уравнение:
13x  5 y  7 ;
100
8. Найдите остаток от деления 2 на 7.
9. Кошке Марусе нужно было покормить и помыть 15 котят. Маруся покормила 8 котят и
помыла 9 котят. После этого выяснилось, что ровно 5 котят покормлены, но не помыты.
Сколько котят не покормлены и не помыты?
10.На игральном кубике общее число точек на любых двух противоположных гранях
равно 7. Мила склеила столбик из 6 таких кубиков и подсчитала общее число точек на
всех наружных гранях. Какое самое большое число она могла получить?
Вариант 12.
1. На сколько нулей оканчивается десятичная запись числа 100!
2.Винни-Пух вышел от Пятачка и отправился домой. Когда он прошёл 160 шагов, его
бросился догонять Пятачок с шариком. Скорость Пятачка в 5 раз больше скорости Винни.
На каком расстоянии от домика Пятачок догонит друга?
3. Натуральное число делится на 12 и имеет ровно 14 различных делителей. Найдите это
число
4.Теперь Ивану вдвое больше лет, чем было Марье тогда, когда ему было столько лет,
сколько Марье теперь. Когда Марье будет столько лет, сколько Ивану теперь, то им
вместе будет 63 года. Определите возраст Ивана и Марьи.
5. Заменив звездочки цифрами в записи ***5 : 11 = **, восстановите равенство.
6.Найдите остаток от деления числа 148100 на 7.
7. Найдите целую и дробную часть числа –48,35.
8. Астролог считает год удачным, если сумма первой и третьей цифры в его номере равна
сумме второй . и четвёртой . Например, 2013 год — удачный . Сколько удачных лет в XXI
веке?
9. Найдите все решения ребуса Я+ОН+ОН+ОН+ОН+ОН+ОН+ОН+ОН=МЫ
10.Диагональ делит четырёхугольник с периметром 31 см на два треугольника с
периметрами 21 см и 30 см. Найдите длину этой диагонали.
Вариант 13.
1. Пять гвардейцев кардинала Ришелье вызвали на поединок четвёрку королевских
мушкетёров (д’Артаньяна, Атоса, Портоса и Арамиса). Помогите мушкетёрам подсчитать
число возможных вариантов первой дуэли.
2. На одной из двух параллельных прямых выбрано 8 точек, а на другой – 6 точек.
Сколько можно построить четырёхугольников с вершинами в данных точках?
4. За столом сидели 5 мальчиков и 6 девочек, а на столе на тарелке лежало несколько
булочек. Каждая из девочек дала по булочке с тарелки каждому знакомому мальчику.
Затем каждый мальчик дал по булочке с тарелки каждой незнакомой ему девочке. После
этого тарелка опустела. Сколько было булочек?
5.Сколько различных делителей имеет число 100000 ?
6. Сколько существует трехзначных чисел, у которых последняя цифра равна
произведению двух первых цифр?
7.Сколько существует двузначных чисел, делящихся на 5 или на 9, но не делящихся на 45?
8. Две стороны треугольника равны соответственно 7 см и 4 см. Чему может быть равна
третья сторона (укажите все возможности)?
9. Гриша из всех цифр составил десятизначное нечётное число. При этом сумма любых
двух соседних цифр меньше десяти. Какое число составил Гриша?
10. После того, как Наташа съела половину персиков из банки, уровень компота
понизился на одну треть. На какую часть от полученного уровня понизится уровень
компота, если съесть половину оставшихся персиков?
Вариант 14.
1.Одно положительное число разделили на другое. Найдите частное, если известно, что
оно в 4 раза больше делимого и в 8 раз больше делителя.
2. Ученик выполняет тест, состоящий из 10 вопросов, каждый из которых имеет два
варианта ответов – “верно” или “неверно’. Сколько может получиться разных вариантов
ответа на весь тест?
3.Отец и сын катаются на коньках по кругу. Время от времени отец обгоняет сына. После
того, как сын переменил направление своего движения на противоположное, они стали
встречаться в пять раз чаще. Во сколько раз отец бегает быстрее сына?
4.Два поезда едут навстречу друг другу: первый со скоростью 50 км/ч, второй – 58 км/ч.
Пассажир, сидевший в первом поезде видел второй поезд в течение 13 секунд. Какова
длина второго поезда?
5.Известно, что n=2a3b, где a≥1, b≥1. Число n2 имеет 15 различных натуральных
делителей. Сколько таких делителей имеет число n3?
6. Какое число надо вычесть из числителя дроби 1234/8747 и прибавить к знаменателю,
чтобы после сокращения получилась 1/8.
7. Число а при делении на 20 дает остаток 2. Какие остатки возможны при делении этого
числа на 8?
8. Придумайте три различных числа, сумма которых делится на каждое из них.
9. Петя сосчитал числа 1 · 2, 2 · 3, 3 · 4, …, 998 · 999, 999 · 1000. У скольких из них
последние две цифры — нули?
10. В школе прошли три олимпиады. Оказалось, что в каждой из них участвовало по 50
человек. Причем, 60 человек приходило только на одну олимпиаду, а 30 человек - ровно
на две. Сколько человек приняло участие во всех трех олимпиадах?
Вариант 15
1. В сенате 100 человек. Каждый из них – либо честный, либо продажный. Известно, что
из любых трёх сенаторов хотя бы один продажен. Каково наибольшее возможное
количество честных сенаторов?
2. На одной из двух параллельных прямых выбрано 8 точек, а на другой – 6 точек.
Сколько можно построить отрезков, соединяющих две точки, лежащие на разных
прямых?
3. Когда сумму цифр двузначного числа сложили с ее квадратом, то получили данное
число. Найдите это число. Необходимо указать все возможные варианты.
4. Решите ребус: ЧАЙ:АЙ=5.
5. По контракту продавцу кваса причитается 400 руб. за каждый отработанный день, а за
каждый неотработанный день с него взыскивается 100 руб. Через 30 дней продавец узнал,
что ему ничего не причитается. Сколько дней работал продавец в течении 30 дней?
6. Между 9 и 9.15 утра киоскер продал 2 журнала и 5 газет на общую сумму 250 рублей.
Между 9.15 и 9.30 он продал еще 5 таких журналов и 2 таких же газеты на сумму 520
рублей. Сколько рублей стоит журнал?
7. Решите в целых числах уравнение:
 2 x  y  5x  3 y   7
8. Найдите p, если: p, p + 10, p + 14 – простые числа.
9.У Феди есть карточки с цифрами 1, 2, 3 и 4 — по две с каждой цифрой. Он хочет
сложить из них число так, чтобы между двумя единицами была одна цифра, между
двойками — две цифры, между тройками — три, а между четвёрками — четыре. Укажите
какое-нибудь число, которое может получить Федя.
10. Винни-Пух и Пятачок играют в слова. Каждый из них составил несколько слов. Они
по очереди называют по одному слову, не повторяя уже названные. Проигрывает тот, кто
не сможет назвать слово. Начинал Винни-Пух, который придумал 11 слов, а выиграл
Пятачок. Какое наименьшее количество слов могло быть у Пятачка?
Вариант 16.
1. В одном карельском селе каждый житель говорит или по-карельски, или по-русски, или
на обоих языках. 653 жителей села говорят по-карельски, 912– по-русски, причем 435
человек говорят на обоих языках. Сколько жителей в этом селе?
2. Сколько слов можно составить, переставляя буквы в имени СНЕЖАННА?
3. Руслан и Эмиль бегают по круговой дорожке вокруг стадиона. Эмиль пробежал один
круг, а Руслан – 10 кругов. Сколько раз за это время Руслан обогнал Эмиля?
4.Сумма всех трехзначных чисел, дающих один и тот же остаток при делении на 19, равна
25803. Чему равен остаток?
5.Сколько процентов 8 процентов составляют от 40 процентов?
6. Сколько существует таких делящихся на 3 трёхзначных чисел, у каждого из которых
все цифры различны.
7. Решить в целых числах уравнение: 21x+15y=7.
8. Найдите последнюю цифру числа 20072006.
9. Из двух одинаковых железных проволок кузнец сковал по одной цепи. Первая содержит
80 одинаковых звеньев, а вторая — 100. Каждое звено первой цепи на 5 граммов тяжелее
каждого звена второй цепи. Какова была масса каждой проволоки?
10. Сумма двух натуральных чисел равна 1244. Если в конце первого приписать 3, а в
конце второго отбросить 2, то числа окажутся равными. Найти эти числа.
В
1
2
15
32
90,15,75
20
16 7n+3
49,
49,94
106
3
90,18,72
В
2
552
2или
6
110
1999
6
24 5
3
91
10
В
3
3
3419
163
27
9
50
20 16
9
3
6,7,8,9
10
В
4
7,5
12
4548
6
3456
24к+
5
18 2
30
58
24
В
5
10
12
3
9876543
4054
182
8
7
100
00
10799998
0
В
6
3,6
1048
576
5
2 127/128
9
0
12,42,
90
530
3
36
58
5
В
7
81
104
6
144
12
40 12
2
20
2.07.13 в
12-00
20
В
8
3
19
20
9,1
12
10 Х=25t y=5-13t
48
58
12
1
В
9
2021
055
15
32
5
20
15 3
2
25
10
В
10
30
или
22,5
42
3528
5
1999
000
96 26
8
11
4104
16
0
В
11
7140
120
14
1
36
27 X=14
+35t
y=35+
13t
2
1
96
В
12
24
200
192
28 и 21
95
1
В
13
20
420
30
7
36
32 24
От3 453627180
до1 9
1
0,2
5
В
14
2
1024
1,5
390
28
12 2 и 6
5
1,2,
3
39
10
В
15
2
48
12,42
,90
125:25=5 37
5:75=5
250:50=5
6
10 (-4,9)
0 и (4,9)
3
41312432
23421314
13
20
22 Реш
8 нет
9
В
16
1130
3360
9
-49 и
0,65
8
0+12+…+
12=96
10
6
2
12
32
и
12
Материалы игр, проводимых в единый день матбоёв
29 июля 2013 года.
Условия немного отличаются и зависят от групп проведения.
1вариант.
1. Решите в натуральных числах уравнение n²+8=m!.
2. Даны четыре попарно различных положительных числа a, b, c и d. Каждую минуту
эти числа одновременно заменяются на a+b+c–d, a+c+d–b, a+b+d–c и b+c+d–a.
Обязательно ли через некоторое время появится отрицательное число?
3. Восемь хоккейных команд провели чемпионат: каждые две команды сыграли ровно
один матч. За победу давалось 2 очка, за ничью – 1 очко, за поражение – 0 очков.
Оказалось, что ровно семь команд поделили второе место. Сколько очков могла
набрать команда-победительница? (Найдите все варианты)
4. С многоугольником разрешено проделывать следующую операцию. Если
многоугольник делится отрезком AB на два многоугольника, то один из этих
многоугольников можно отразить симметрично относительно серединного
5.
6.
7.
8.
перпендикуляра к отрезку AB. (Операция разрешается только в том случае, когда в
результате получается несамопересекающийся многоугольник.) Можно ли путем
нескольких таким операций получить из квадрата правильный треугольник?
Вещественные числа x, y и z таковы, что xyz=1. Докажите, что (2+x)(2+y)(2+z)≤
27.
Фигура «сфинкс» состоит из 6 правильных треугольников со стороной 1
(см. рис.). Можно ли правильный треугольник со стороной 30 разрезать
на сфинксов? (Фигурки можно поворачивать и переворачивать.)
На Петиной чаше двухчашечных весов лежат гири весом 1 г, 3 г, ..., 2001 г, а на
Васиной чаше – 2 г, 4 г, ..., 2000 г. Первым ходит Петя – он убирает в некотором
порядке по одной гире со своей чаши до тех пор, пока она не станет легче Васиной.
Потом Вася убирает по одной гире со своей чаши до тех пор, пока она не станет
легче Петиной. Затем опять ходит Петя, потом Вася, и так далее. Выигрывает тот,
кто первым сможет убрать все гири со своей чаши. Кто может выиграть
независимо от игры соперника?
Каждая клетка тетрадного листа закрашена в какой-нибудь цвет. Известно, что в
каждом L-тетрамино все клетки разного цвета. При каком наименьшем количестве
цветов это возможно? (L-тетрамино – это фигурка из четырёх клеток в форме
буквы L или Г, возможно, повёрнутая.)
2 вариант.
1. Бесконечная последовательность натуральных чисел a0, a1..., an, ... такова, что a0=1,
а an2>an-1*an+1 (при n>0). Докажите, что ak>k при всех натуральных k.
2. В выпуклом четырехугольнике ABCD точки P, Q, R, S — середины сторон AB, BC,
CD, DA соответственно, а точки K, L — середины диагоналей AC и BD
соответственно. Диагонали пересекаются в точке O. Внутри четырехугольника
нашлась такая точка X, что OKXL — параллелограмм. Докажите, что площади
четырехугольников XSAP, XPBQ, XQCR и XRDS равны.
3. Рассмотрим множество из 20 целых чисел {a1, a2, …, a10, -a1, -a2, …, -a10}.
Докажите, что из этого множества можно выбрать непустое подмножество S такое,
что никакие два числа ai и -ai не могут оба лежать в S, и сумма всех чисел из S
делится на 1001.
4. Решите в натуральных числах уравнение n2+8=m!
5. С многоугольником разрешено проделывать следующую операцию. Если
многоугольник делится отрезком AB на два многоугольника, то один из этих
многоугольников можно отразить симметрично относительно серединного
перпендикуляра к отрезку AB. (Операция разрешается только в том случае, когда в
результате получается несамопересекающийся многоугольник.) Можно ли путем
нескольких таким операций получить из квадрата правильный треугольник?
6. Хроматическое число графа G равно k. Известно, что существует правильная
раскраска вершин графа G такая, что вершин каждого цвета хотя бы две. Докажите,
что существует такая раскраска в k цветов. (Хроматическим числом графа
называется наименьшее число цветов, в которое можно правильно раскрасить
вершины этого графа.)
7. Положительные числа x, y и z таковы, что xyz=1. Докажите, что (2+x)(2+y)(2+z) ≤
27.
8. Фигура «сфинкс» состоит из 6 правильных треугольников со стороной 1 (см. рис).
Можно ли правильный треугольник со стороной 30 разрезать на сфинксов?
(Фигурки можно поворачивать и переворачивать.)
ПРАВИЛА
МАТЕМАТИЧЕСКИХ
МАТЕМАТИЧЕСКИЙ
ИГР.
БОЙ.
Матбой - это соревнование двух команд в решении нестандартных задач, подобранных
жюри, в умении отвечать решения у доски и в умении проверять чужие решения.
Команды получают одинаковые задачи, и решают их в разных помещениях заданное
время, потом собираются вместе для проверки решений. Таким образом, матбой состоит
из двух частей: решения задач и собственно боя.
Чтобы определить, кто какую задачу будет отвечать, команды делают "вызовы": одна
называет номер задачи, решение которой она хочет услышать, а другая сообщает, принят
ли вызов. Обычно команды вызывают друг друга по очереди.
Если вызванная команда хочет отвечать, то она выставляет докладчика, а другая команда
- оппонента для проверки решения. Командам могут даваться минутные перерывы для
помощи докладчику или оппоненту.
Если решение задачи принято жюри, то переходят к обсуждению другой задачи, а если не
принято, то см. "перемена ролей" и "корректность вызова".
Если вызванная команда отказалась отвечать, то вызывавшая команда должна сама
рассказать решение задачи. При этом если оппонент докажет, что у докладчика нет
решения, то вызов считается некорректным. Тогда вызывавшая команда должна
повторить вызов.
Команда может отказаться делать очередной вызов (если у нее не осталось решенных
задач и она не хочет делать некорректный вызов). Тогда другая команда получает право
рассказать решения любых задач, оставшихся не разобранными.
После каждого выступления жюри дает командам очки, как за доклад, так и за
оппонирование.
Решение задач
Есть джентльменское правило: прежде, чем решать задачи, команды сообщают жюри все
задачи, решения которых им известны (матбой - это не клуб знатоков). Жюри исключает
или заменяет эти задачи (предварительно проверив, что идея решения действительно
известна).
Представитель жюри регулярно посещает команды и отвечает на вопросы по условиям
задач. При этом каждое уточнение условий, данное одной команде, сразу же должно
сообщаться и другой команде.
Жюри не должно давать информации о трудности задач. В процессе решения задач и во
время боя команды не должны общаться и знать количество решенных задач у
соперников.
Ход боя
Когда время на решение задач истекло, команды и жюри собираются вместе.
Существуют ограничения на общение участников,
которые показаны на схеме (например, оппонент может
общаться только с докладчиком и с жюри, а капитан только со своей командой и с жюри).
Конкурс капитанов
Перед началом обсуждения необходимо определить, какая команда первой будет делать
вызов. Для этого проводится конкурс капитанов. Капитаны выходят к доске и получают
достаточно простую задачу на сообразительность, в которой требуется дать только ответ,
или игру, в которой не видно простой выигрышной стратегии (при этом капитанов
спрашивают, кто хочет начать игру или быть вторым. Кто раньше ответит - определит
очередность).
Конкурс кончается, когда один из капитанов даст ответ или победит в игре. Если ответ
верен, то капитан победил, а если неверен, то победил другой капитан.
Капитан, победивший в конкурсе, сообщает, какая команда сделает первый вызов.
Вызов
Капитан вызывающей команды сообщает номер задачи, решение которой команда хочет
услышать, а другая команда отвечает, принят ли вызов.
Если вызванная команда хочет отвечать, то она сообщает, что вызов принят и выставляет
докладчика, а вызывавшая команда - оппонента для проверки решения.
Если вызванная команда отказалась отвечать, то вызывавшая команда должна сама
предъявить решение (выставить докладчика, а другая команда - оппонента). В этом случае
говорят, что происходит проверка корректности вызова.
Докладчик и оппонент
В идеале: сначала докладчик рассказывает решение, затем оппонент задает вопросы,
после чего оппонент сообщает свое мнение о решении (например, "решение не
принимается, т.к. такой-то факт не доказан, а на такой-то вопрос не получено
удовлетворительного ответа"). И только после этого свои вопросы докладчику задает
жюри.
В процессе доклада оппонент и жюри стремятся не прерывать докладчика и пользуются
лишь выражениями типа: "это очевидно, можно не доказывать", "повторите, пожалуйста,
этот момент".
Докладчик может не отвечать на вопросы оппонента во время доклада, но по требованию
оппонента или жюри должен дать план решения.
Оппонент не должен требовать доказательства утверждений из школьной программы или
круга "известных" фактов. В спорных случаях вопрос решает жюри.
Время на обдумывание вопросов у доски I минута (оппоненту - чтобы задать, докладчику чтобы ответить).
Команды могут помогать докладчику и оппоненту только во время минутного перерыва
(соперники тоже пользуются этой минутой). Во время своего минутного перерыва можно
заменить докладчика или оппонента (при этом учитывается выход к доске).
Если за минуту, данную на обдумывание вопроса, который жюри считает существенным,
докладчик не подготовил ответ, и команда не взяла минутный перерыв, то считается, что в
решении есть пробел ("дырка").
Перемена ролей
Если в решении имеются "дырки", обнаруженные оппонентом, то, после того как жюри
задаст докладчику свои вопросы, вызывавшая команда получает право (но не обязана)
"залатать" эти "дырки" (но она не имеет права "латать" дыры, найденные не оппонентом,
а жюри; тем более она не имеет права рассказывать свое решение). Происходит перемена
ролей - теперь докладчик становится оппонентом, а оппонент - докладчиком. При этом
"новый оппонент" (бывший докладчик) может получить очки за оппонирование, но
повторной перемены ролей произойти не может.
Только в том случае, когда оппонент смог доказать, что у докладчика полностью
отсутствует решение (и жюри согласно с этим), т.е. что имеется "дырка" размером в
полное решение, вызывавшая команда получает право рассказывать свое решение происходит перемена ролей.
Если оппонент не нашел пробелов и его команда не взяла минутный перерыв, то он и его
команда в обсуждении задачи больше не участвуют.
Во время перемены ролей можно заменить бывших докладчика или оппонента (при этом
учитывается выход к доске).
Корректность вызова
Если вызов принят, то вопроса о его корректности не ставится (иногда говорят: "принятый
вызов всегда корректен").
Если вызов не принят, то вызывавшая команда должна сама рассказать решение, и здесь
возможны два случая:
вызывавшая команда не стала отвечать. Тогда вызов "автоматически" считается
некорректным;
вызывавшая команда выставила докладчика. Тогда происходит обычное обсуждение
задачи докладчиком (от вызывавшей команды) и оппонентом (от вызванной) со
следующими особенностями:
а).перемена ролей произойти не может -т.к. вызванная команда уже отказалась
рассказывать решение;
б).решающее значение имеет ответ оппонента на традиционный вопрос жюри
"принимается ли решение?". Если решение нe принимается, то оппонент должен строго
обосновать свои претензии к решению.
Вызов признается некорректным в двух случаях:
1.
вызывавшая команда не стала отвечать;
2.
вызывавшая команда выставила докладчика, но рассказала менее половины
решения (т.е. не более чем на 6 баллов), и при этом оппонент не принял решения (если
оппонент принял решение, не разглядев в нем "липу", то вызов считается корректным).
При некорректном вызове оппонент получает 6 очков, а вызывавшая команда - до 6
очков за верные идеи и должна повторить вызов.
Отказ делать вызов
Если у команды не осталось решенных задач, то она отказывается делать вызов (чтобы
избежать некорректного вызова). Тогда другая команда получает право рассказать все
оставшиеся у нее решения.
После отказа от вызова команда до конца боя теряет право рассказывать решения задач и
становится "вечным оппонентом", т.е. может получать очки только за оппонирование.
Начисление очков
Каждая задача стоит 12 очков (чтобы не сообщать трудность задач). Эти очки
распределяются между докладчиком, оппонентом и жюри (жюри достается остаток от 12
очков).
Очки даются как за положительный вклад в решение задачи, так и за нахождение ошибок
и пробелов в решении. За чистое решение задачи дается 12 очков, а за "полное"
оппонирование - 6 очков (если оппонент показал, что у докладчика совсем нет
положительных результатов).
Сначала жюри определяет стоимость {в очках) рассказанной докладчиком части (он и
получает эти очки) и стоимость каждой "дырки" в решении. За каждую найденную
"дырку" оппонент получает половину стоимости этой "дырки" (если "дырку" нашло
жюри, то оно и получает очки). Вторую половину стоимости этой "дырки" получит тот,
кто ее "залатает" - докладчик (если ответит на вопрос оппонента), оппонент (при перемене
ролей) или жюри (если никто "дырку" не закроет). При перемене ролей для подсчета
очков применяют те же самые рассуждения.
Пример:
Докладчик рассказал решение. Оппонент нашел дырку-1. Жюри задало вопросы докладчику и
нашло еще две дырки: дырку-2 и дырку-3, причем дырку-2 докладчик залатал у доски.
Жюри разделило очки так: рассказанная часть - 2 очка, дырка-1 - б очков, дыр-ка-2 - 2 очка,
дырка-3 - 2 очка.
Оппонент получил право рассказывать дырку-1 - т.е. произошла перемена ролей (стоит эта
дырка 6 очков, 3 из которых уже получил оппонент; значит, сейчас разыгрывается 3 очка).
При этом "новый оппонент" (бывший докладчик) нашел в его рассказе дырку-4.
Жюри оценило так: рассказанная часть -1 очко (из 3), дырка-4 - 2 очка (из 3).
Общий счет:
Докладчик: 2 (рассказанная часть) + / (половина стоимости дырки-2 - т.к. он ее залатал у
доски) + 1 (половина стоимости дырки-4 - т.к. он ее нашел, находясь в роли оппонента) = 4
очка.
Оппонент: 3 (половина стоимости дырки-1) + / (рассказанная им часть при перемене
ролей) -4 очка.
Жюри: оставшиеся 4 очка.
За красивое решение или красивое оппонирование жюри может дать одно премиальное
очко (оно не входит в те 12 очков).
Жюри дает очки гласно, т.е. объясняет, за что они даны или сняты.
Жюри может оштрафовать команду на очко за шум, за неэтичное поведение (после
предупреждения). За подсказку штраф может быть больше с прекращением дискуссии по
задаче и удалением подсказавшего.
Если остается время, жюри может выслушать более красивые решения и давать за них
премиальные очки.
Итоги
После каждого вызова жюри сообщает, поясняет и записывает, сколько очков получила
каждая команда. Жюри ведет протокол матбоя в виде таблицы, в которой указываются:
фамилии выступающих, номер обсуждаемой задачи, направление вызова, взятые
минутные перерывы и. количество очков, полученных командами и оставшихся у жюри.
На доске рисуется упрощенная таблица, без указания фамилий.
После боя очки у каждой команды и у жюри складываются (количество очков,
оставшихся у жюри, характеризует трудность задач и силу команд).
Если разность очков команд не превышает трех, то засчитывается ничья.
Если остается время, то жюри рассказывает решения нерешенных во время матбоя задач
или показывает более удачные решения.
Статус жюри
Жюри является верховным толкователем правил матбоя. Если ситуация правилами не
предусмотрена, жюри принимает решение по своему усмотрению. Решение жюри является
обязательным для команд,
Жюри может снять вопрос оппонента (если вопрос не по существу), прекратить доклад
или оппонирование (если дискуссия затягивается). Во всех подобных случаях жюри
обосновывает свое решение.
Всякие соображений по уже разобранным задачам жюри рассматривает после боя. Задним
числом счет изменять нельзя.
Статус ведущего
Ведущий обязан следить за порядком обсуждения, в частности:
предоставлять слово докладчику;
объявлять о завершении доклада и переходе к обсуждению;
объявлять начало и конец минутного перерыва, взятого командой;
фиксировать вопросы оппонента и ответы на них докладчика (например, спрашивая
оппонента: "Вы удовлетворены ответом?" и т.д.);
•фиксировать мнение оппонента о докладе ("Решение принимается?" или -если решение
не принимается - "С чем Вы не согласны в решении?");
объявлять о завершении обсуждения и переходе к заданию вопросов жюри докладчику;
не позволять оппонентам перебивать докладчика;
не позволять обсуждению выходить за рамки научной дискуссии;
объявлять распределение очков за задачу, поясняя, за что они даны или сняты.
Статус капитана
Капитан отвечает перед командой за организацию решения задач, подготовку
докладчиков и оппонентов, тактику ведения боя.
Капитан является представителем команды по всем оргвопросам: только он делает вызов,
берет минутный перерыв, общается с жюри (если капитан выходит к доске, то он
оставляет заместителя).
Капитан заранее определяет, кто будет докладчиком и кто оппонентом по каждой задаче,
решает взять или отдать первый вызов.
Договорные условия
1. Предельное число выходов к доске одного человека - 2 (не считая конкурса капитанов).
2. Число минутных перерывов - 3.
З.Примерное время на доклад (после которого жюри решает: дать еще время или передать
слово оппоненту) - 10 минут (без учета времени ответов на вопросы оппонента).
4.Примерное время на дискуссию - 7 минут (без учета времени на рассказ решения
докладчиком).
5.Какую разницу очков считать ничейной - 3.
б.Можно ли пользоваться литературой и калькуляторами во время решения задач - да.
7.Можно ли выходить к доске с записанным решением - да.
Памятка жюри
1 .Жюри должно знать решения всех задач.
2.Жюри должно помнить, что своими вопросами оно помогает докладчику доработать
решение у доски, а вмешиваясь в диалог, "ест хлеб" оппонента.
З.Если жюри (после вопросов оппонента) видит пробел в решении, то оно должно
проверить, может ли докладчик его закрыть.
4.Сначала обсуждаются оргвыводы (наличие решения, достаточность оппонирования и
т.д.), затем обсуждаются очки.
5.Если докладчик несет полную чушь, то лучше всего попросить предъявить план
решения - у "лапши" не бывает плана. Но это надо делать после вопросов оппонента - см.
памятку 2.
б.Если жюри не может быстро разобраться в решении, то в целях экономии времени и сил
участников с согласия капитанов жюри может выделить своего представителя, который
пойдет разбираться с докладчиком и оппонентом в другое помещение. При этом бой
продолжается, а очки по задаче начисляются позднее. (Это возможно, если нет проверки
корректности вызова).
7.Желательно в течение боя в аналогичных ситуациях принимать аналогичные решения
(правило прецедента).
ДОМИНО.
Правила математической игры «Домино»:
1. В игре могут участвовать до 10 команд (по 4 человека в каждой команде) на один
комплект задач. Игра идёт в течение 3-4 часов, о чём сообщается заранее.
2. Протокол игры ведётся жюри с выводом на экран текущих результатов через
мультимедийный проектор.
3. Каждая из 28 задач имеет свою стоимость согласно распределению баллов на
доминошках (0-0, 0-1, 0-2, …, 4-6, 5-5, 5-6, 6-6).
4. Каждая команда получает изначально две задачи с суммарной стоимостью не более 8
баллов случайным образом из банка задач, который находится у жюри (каждая задача
каждого комплекта - в одном экземпляре). После этого при сдаче ответа вместе с
условием задачи команда самостоятельно берёт себе новую задачу. При этом у команды
на руках всегда две задачи.
5. На каждую задачу (кроме 0-0) команда может дать ответ только два раза.
6. Если сразу даны верный ответ или решение, то команда получает полное суммарное
количество баллов соответствующей доминошки. Если же с первого раза даны неверный
ответ или решение, то в протокол ставится 0 баллов, и со второй попытки (после взятия
этой задачи в будущем) за верное решение команда сможет получить только большую
часть баллов доминошки. После двух неудачных попыток задача больше не принимается,
а команда наказывается штрафом, равным меньшей части баллов доминошки.
Невозможность в будущем решать командой задачу со штрафом в 0 баллов отмечается в
протоколе жёлтым цветом (карточкой).
7. Задача 0-0 при верном решении с первой попытки даёт 10 баллов, если же решение
неверное, то задача больше не принимается, по ней команда получает 0 баллов и жёлтую
карточку.
8. Если команда не может решить задачу или не хочет давать по ней ответ, то она может
её «сбросить», т.е. сдать в жюри без получения полагающегося штрафа (в этом случае
команда должна сдать листочек, на котором ответ не указывается, записывается слово
«сброс» или ставится прочерк). При этом команда может взять себе эту задачу в будущем,
если по ней у команды пока ещё 0 баллов и нет жёлтой карточки. В случае первой
попытки при сбросе команда получает 0 баллов, в случае второй попытки команде
оставляются 0 баллов и даётся желтая карточка.
9. Если команда ошибочно взяла задачу, которую решала ранее и уже получила по ней
соответствующий ненулевой балл или жёлтую карточку, то она наказывается одним
штрафным баллом, который выставляется в графу «штраф». Сдаёт эту задачу в жюри и
берёт себе новую.
10. Ответ на задачу команда указывает на специальном бланке. В случае неверного
оформления листка ответа (отсутствие названия команды, цены задачи, эмблемы-логотипа
на задней стороне) команда наказывается штрафным баллом.
11. Если во время или по окончании игры в ответах жюри обнаружится ошибка, то
команда, сдавшая
правильный ответ, получает удвоенное количество баллов, полагавшихся ей за
правильный ответ в момент его сдачи. Баллы за подобную задачу у команд, которым был
засчитан неверный ответ, обнуляются.
12. Игра для команды прекращается либо по окончании отведённого на неё времени, либо
после того, как командой разобраны все 28 задач.
13. Команды по итогам игры занимают места по убыванию количества набранных ими
баллов.
КАРУСЕЛЬ.
Математическая карусель – это командное соревнования по решению задач. Побеждает
в нем команда, набравшая наибольшее число очков. Задачи решаются на двух рубежах –
исходном и зачетном, но очки начисляются только за задачи, решенные на зачетном
рубеже. В начале игры все члены команды располагаются на исходном рубеже, причем им
присвоены номера от 1 до 6. По сигналу ведущего команды получают задачу и начинают
ее решать. Если команда считает, что задача решена, ее представитель, имеющий номер 1,
предъявляет решение судье. Если оно верное, игрок №1 переходит на зачетный рубеж и
получает задачу там, а члены команды, оставшиеся на исходном рубеже, тоже получают
новую задачу. В дальнейшем члены команды, находящиеся на исходном и зачетном
рубежах, решают разные задачи независимо друг от друга.
Чтобы понять следующую часть правил, надо представить себе, что на каждом рубеже
находящиеся на нем члены команды выстроены в очередь. Перед началом игры на
исходном рубеже они идут в ней в порядке номеров. Если члены команды, находящиеся
на каком-либо из двух рубежей, считают, что они решили очередную задачу, решение
предъявляет судье игрок, стоящий в очереди первым. Если решение правильное, то с
исходного рубежа этот игрок переходит на зачетный, а на зачетном возвращается на свое
место в очереди. Если решение неправильное, то на исходном рубеже игрок возвращается
на свое место в очереди, а с зачетного переходит на исходный. Игрок, перешедший с
одного рубежа на другой, становится в конец очереди. И на исходном, и на зачетном
рубежах команда может в любой момент отказаться от решения задачи. При этом задача
считается нерешенной.
После того, как часть команды, находящаяся на каком-либо из двух рубежей, рассказала
решение очередной задачи или отказалась решать ее дальше, она получает новую задачу.
Если на рубеже в этот момент нет ни одного участника, задача начинает решаться тогда,
когда этот участник там появляется.
За первую верно решенную на зачетном рубеже задачу команда получает 3 балла. Если
команда на зачетном рубеже верно решает несколько задач подряд, то за каждую
следующую задачу она получает на 1 балл больше, чем за предыдущую. Если же
очередная задача решена неверно, то цена следующей задачи зависит от ее цены
следующим образом. Если цена неверно решенной задачи была больше 6 баллов, то
следующая задача стоит 5 баллов. Если цена неверно решенной задачи была 4, 5 или 6
баллов, то следующая задача стоит на балл меньше. Если же неверно решенная задача
стоила 3 балла, то следующая задача тоже стоит 3 балла.
Игра для команды оканчивается, если
а) кончилось время, или
б) кончились задачи на зачетном рубеже, или
в) кончились задачи на исходном рубеже, а на зачетном рубеже нет ни одного игрока.
Время игры, количество исходных и зачетных задач оговаривается заранее.
МАТЕМАТИЧЕСКАЯ ДУЭЛЬ.
1) Перед дуэлью учащиеся разбиваются на пары (по своему выбору или на выбор учителя
или ведущего).
2) Каждый учащийся получает карточку с 10 задачами из общего списка. Задачи
распределяются случайно.
3) В первый час учащиеся решают свои задачи.
4) В следующий за этим час учащиеся решают задачи соперника.
6) После этого пары вызываются к доске. Учащиеся записывает на доске только ответы к
задачам соперника.
7) Учащийся может отказаться писать ответы к некоторым задачам.
8) Соперник имеет право либо согласиться с приведенными ответами, либо указать
неправильные, либо привести свой ответ. Соперник имеет право отказаться от любого
упомянутого в этом пункте действия.
9) За каждое правильное действие учащийся получает 1 очко, за каждое неправильное
действие отнимается 1 очко.
10) По итогам математической дуэли победитель получает столько баллов, какова разница
в счете.
Download